SlideShare una empresa de Scribd logo
1 de 128
Descargar para leer sin conexión
5




                            Uma palavra aos alunos e professores
                                                                                           9




             Uma palavra aos alunos e professores

     O Banco de Quest˜es (BQ) foi concebido para divulgar nas escolas da rede p´blica material
                       o                                                       u
de competi¸˜es de Matem´tica, nacionais ou internacionais. Por isso grande parte do conte´do
            co              a                                                            u
n˜o ´ original, s˜o quest˜es dessas competi¸˜es ou de prepara¸˜o para elas encontradas em
  a e            a         o                co                 ca
diversos sites e apostilas. Aproveitamos para agradecer a todos que mantˆm esses sites com
                                                                           e
livre acesso pela grande contribui¸˜o que d˜o a tantos alunos e professores.
                                  ca       a
    Como temos feito desde 2 005, n˜o nos preocupamos com uniformidade. A cada ano o
                                      a
BQ apresenta formato, quantidade e n´ de dificuldade diferentes dos anos anteriores. A
                                        ıvel
linguagem usada nas solu¸˜es ´ bastante informal mas sem comprometer o rigor matem´tico.
                         co e                                                          a
O BQ n˜o ´ um livro did´tico e por isso continuamos a produzi-lo de forma bastante artesanal.
       a e             a
   Incentivamos alunos e professores a procurar solu¸˜es diferentes das aqui apresentadas,
                                                     co
com certeza elas existem e podem ser mais interessantes.
   Por solicita¸˜o de muitos alunos, retomamos esse ano a sess˜o Desafios aonde os problemas
               ca                                             a
requerem mais paciˆncia, mais tempo e mais aten¸˜o. Aproveitamos para informar que temos
                    e                            ca
agora no site da OBMEP (www.obmep.org.br ) a sess˜o “Problemas da 15na” com material
                                                     a
muito instigante e desafiador para aqueles que gostam de “quebrar a cabe¸a” com problemas
                                                                         c
de Matem´tica.
          a
   Os problemas est˜o agrupados em trˆs n´
                    a                   e ıveis conforme ´ feito nas provas da OBMEP, mas
                                                         e
muitos s˜o interessantes para todos os alunos.
        a
   Sugest˜es quaisquer (por exemplo, de solu¸˜es diferentes) ou cr´
          o                                 co                    ıticas ser˜o bem recebidas
                                                                            a
no email: contato@obmep.org.br
   Desejamos que esse Banco de Quest˜es proporcione a todos bons momentos de reflex˜o e
                                    o                                             a
descobertas.

                                                             Dire¸˜o Acadˆmica da OBMEP
                                                                 ca      e




                                       OBMEP 2009                                              i
5




                            Uma palavra aos alunos e professores
           9




     Organizado por:
                      • Suely Druck (UFF)
                      • Maria Elasir Seabra Gomes (UFMG)
     Com a colabora¸˜o de:
                   ca
                      • Ana Catarina P. Hellmeister (USP/SP)
                      • F´bio Brochero (UFMG)
                         a
                      • Francisco Dutenhefner (UFMG)




     Texto j´ revisado pela nova ortografia.
            a




ii                                      OBMEP 2009
Conte´do
     u

Uma palavra aos alunos e professores                                                                                                                                                    i

N´ 1
 ıvel                                                                                                                                                                                   1
   Lista   1 .     .   .   .   .   .   .   .   .   .   .   .   .   .   .   .   .   .   .   .   .   .   .   .   .   .   .   .   .   .   .   .   .   .   .   .   .   .   .   .   .   .    1
   Lista   2 .     .   .   .   .   .   .   .   .   .   .   .   .   .   .   .   .   .   .   .   .   .   .   .   .   .   .   .   .   .   .   .   .   .   .   .   .   .   .   .   .   .    2
   Lista   3 .     .   .   .   .   .   .   .   .   .   .   .   .   .   .   .   .   .   .   .   .   .   .   .   .   .   .   .   .   .   .   .   .   .   .   .   .   .   .   .   .   .    3
   Lista   4 .     .   .   .   .   .   .   .   .   .   .   .   .   .   .   .   .   .   .   .   .   .   .   .   .   .   .   .   .   .   .   .   .   .   .   .   .   .   .   .   .   .    4
   Lista   5 .     .   .   .   .   .   .   .   .   .   .   .   .   .   .   .   .   .   .   .   .   .   .   .   .   .   .   .   .   .   .   .   .   .   .   .   .   .   .   .   .   .    5
   Lista   6 .     .   .   .   .   .   .   .   .   .   .   .   .   .   .   .   .   .   .   .   .   .   .   .   .   .   .   .   .   .   .   .   .   .   .   .   .   .   .   .   .   .    6
   Lista   7 .     .   .   .   .   .   .   .   .   .   .   .   .   .   .   .   .   .   .   .   .   .   .   .   .   .   .   .   .   .   .   .   .   .   .   .   .   .   .   .   .   .    7
   Lista   8 .     .   .   .   .   .   .   .   .   .   .   .   .   .   .   .   .   .   .   .   .   .   .   .   .   .   .   .   .   .   .   .   .   .   .   .   .   .   .   .   .   .    8
   Lista   9 .     .   .   .   .   .   .   .   .   .   .   .   .   .   .   .   .   .   .   .   .   .   .   .   .   .   .   .   .   .   .   .   .   .   .   .   .   .   .   .   .   .    9
   Lista   10      .   .   .   .   .   .   .   .   .   .   .   .   .   .   .   .   .   .   .   .   .   .   .   .   .   .   .   .   .   .   .   .   .   .   .   .   .   .   .   .   .   10

N´ 2
 ıvel                                                                                                                                                                                  11
   Lista   1 .     .   .   .   .   .   .   .   .   .   .   .   .   .   .   .   .   .   .   .   .   .   .   .   .   .   .   .   .   .   .   .   .   .   .   .   .   .   .   .   .   .   11
   Lista   2 .     .   .   .   .   .   .   .   .   .   .   .   .   .   .   .   .   .   .   .   .   .   .   .   .   .   .   .   .   .   .   .   .   .   .   .   .   .   .   .   .   .   12
   Lista   3 .     .   .   .   .   .   .   .   .   .   .   .   .   .   .   .   .   .   .   .   .   .   .   .   .   .   .   .   .   .   .   .   .   .   .   .   .   .   .   .   .   .   13
   Lista   4 .     .   .   .   .   .   .   .   .   .   .   .   .   .   .   .   .   .   .   .   .   .   .   .   .   .   .   .   .   .   .   .   .   .   .   .   .   .   .   .   .   .   14
   Lista   5 .     .   .   .   .   .   .   .   .   .   .   .   .   .   .   .   .   .   .   .   .   .   .   .   .   .   .   .   .   .   .   .   .   .   .   .   .   .   .   .   .   .   15
   Lista   6 .     .   .   .   .   .   .   .   .   .   .   .   .   .   .   .   .   .   .   .   .   .   .   .   .   .   .   .   .   .   .   .   .   .   .   .   .   .   .   .   .   .   16
   Lista   7 .     .   .   .   .   .   .   .   .   .   .   .   .   .   .   .   .   .   .   .   .   .   .   .   .   .   .   .   .   .   .   .   .   .   .   .   .   .   .   .   .   .   17
   Lista   8 .     .   .   .   .   .   .   .   .   .   .   .   .   .   .   .   .   .   .   .   .   .   .   .   .   .   .   .   .   .   .   .   .   .   .   .   .   .   .   .   .   .   18
   Lista   9 .     .   .   .   .   .   .   .   .   .   .   .   .   .   .   .   .   .   .   .   .   .   .   .   .   .   .   .   .   .   .   .   .   .   .   .   .   .   .   .   .   .   19
   Lista   10      .   .   .   .   .   .   .   .   .   .   .   .   .   .   .   .   .   .   .   .   .   .   .   .   .   .   .   .   .   .   .   .   .   .   .   .   .   .   .   .   .   20

N´ 3
 ıvel                                                                                                                                                                                  21
   Lista   1   .   .   .   .   .   .   .   .   .   .   .   .   .   .   .   .   .   .   .   .   .   .   .   .   .   .   .   .   .   .   .   .   .   .   .   .   .   .   .   .   .   .   21
   Lista   2   .   .   .   .   .   .   .   .   .   .   .   .   .   .   .   .   .   .   .   .   .   .   .   .   .   .   .   .   .   .   .   .   .   .   .   .   .   .   .   .   .   .   22
   Lista   3   .   .   .   .   .   .   .   .   .   .   .   .   .   .   .   .   .   .   .   .   .   .   .   .   .   .   .   .   .   .   .   .   .   .   .   .   .   .   .   .   .   .   23
   Lista   4   .   .   .   .   .   .   .   .   .   .   .   .   .   .   .   .   .   .   .   .   .   .   .   .   .   .   .   .   .   .   .   .   .   .   .   .   .   .   .   .   .   .   25
   Lista   5   .   .   .   .   .   .   .   .   .   .   .   .   .   .   .   .   .   .   .   .   .   .   .   .   .   .   .   .   .   .   .   .   .   .   .   .   .   .   .   .   .   .   26
   Lista   6   .   .   .   .   .   .   .   .   .   .   .   .   .   .   .   .   .   .   .   .   .   .   .   .   .   .   .   .   .   .   .   .   .   .   .   .   .   .   .   .   .   .   27
   Lista   7   .   .   .   .   .   .   .   .   .   .   .   .   .   .   .   .   .   .   .   .   .   .   .   .   .   .   .   .   .   .   .   .   .   .   .   .   .   .   .   .   .   .   28
   Lista   8   .   .   .   .   .   .   .   .   .   .   .   .   .   .   .   .   .   .   .   .   .   .   .   .   .   .   .   .   .   .   .   .   .   .   .   .   .   .   .   .   .   .   29
   Lista   9   .   .   .   .   .   .   .   .   .   .   .   .   .   .   .   .   .   .   .   .   .   .   .   .   .   .   .   .   .   .   .   .   .   .   .   .   .   .   .   .   .   .   30

                                                                                       iii
5




                                     Uma palavra aos alunos e professores
           9




     Lista 10 . . . . . . . . . . . . . . . . . . . . . . . . . . . . . . . . . . . . . . . . . 31

Desafios                                                                                                                                                                  32

Solu¸˜es do N´ 1
    co         ıvel                                                                                                                                                      35
   Lista 1 . . . . . .   .   .   .   .   .   .   .   .   .   .   .   .   .   .   .   .   .   .   .   .   .   .   .   .   .   .   .   .   .   .   .   .   .   .   .   .   35
   Lista 2 . . . . . .   .   .   .   .   .   .   .   .   .   .   .   .   .   .   .   .   .   .   .   .   .   .   .   .   .   .   .   .   .   .   .   .   .   .   .   .   38
   Lista 3 . . . . . .   .   .   .   .   .   .   .   .   .   .   .   .   .   .   .   .   .   .   .   .   .   .   .   .   .   .   .   .   .   .   .   .   .   .   .   .   40
   Lista 4 . . . . . .   .   .   .   .   .   .   .   .   .   .   .   .   .   .   .   .   .   .   .   .   .   .   .   .   .   .   .   .   .   .   .   .   .   .   .   .   43
   Lista 5 . . . . . .   .   .   .   .   .   .   .   .   .   .   .   .   .   .   .   .   .   .   .   .   .   .   .   .   .   .   .   .   .   .   .   .   .   .   .   .   46
   Lista 6 . . . . . .   .   .   .   .   .   .   .   .   .   .   .   .   .   .   .   .   .   .   .   .   .   .   .   .   .   .   .   .   .   .   .   .   .   .   .   .   49
   Lista 7 . . . . . .   .   .   .   .   .   .   .   .   .   .   .   .   .   .   .   .   .   .   .   .   .   .   .   .   .   .   .   .   .   .   .   .   .   .   .   .   53
   Lista 8 . . . . . .   .   .   .   .   .   .   .   .   .   .   .   .   .   .   .   .   .   .   .   .   .   .   .   .   .   .   .   .   .   .   .   .   .   .   .   .   55
   Lista 9 . . . . . .   .   .   .   .   .   .   .   .   .   .   .   .   .   .   .   .   .   .   .   .   .   .   .   .   .   .   .   .   .   .   .   .   .   .   .   .   57
   Lista 10 . . . . .    .   .   .   .   .   .   .   .   .   .   .   .   .   .   .   .   .   .   .   .   .   .   .   .   .   .   .   .   .   .   .   .   .   .   .   .   59

Solu¸˜es do N´ 2
    co         ıvel                                                                                                                                                      61
   Lista 1 . . . . . .   .   .   .   .   .   .   .   .   .   .   .   .   .   .   .   .   .   .   .   .   .   .   .   .   .   .   .   .   .   .   .   .   .   .   .   .   61
   Lista 2 . . . . . .   .   .   .   .   .   .   .   .   .   .   .   .   .   .   .   .   .   .   .   .   .   .   .   .   .   .   .   .   .   .   .   .   .   .   .   .   63
   Lista 3 . . . . . .   .   .   .   .   .   .   .   .   .   .   .   .   .   .   .   .   .   .   .   .   .   .   .   .   .   .   .   .   .   .   .   .   .   .   .   .   66
   Lista 4 . . . . . .   .   .   .   .   .   .   .   .   .   .   .   .   .   .   .   .   .   .   .   .   .   .   .   .   .   .   .   .   .   .   .   .   .   .   .   .   69
   Lista 5 . . . . . .   .   .   .   .   .   .   .   .   .   .   .   .   .   .   .   .   .   .   .   .   .   .   .   .   .   .   .   .   .   .   .   .   .   .   .   .   72
   Lista 6 . . . . . .   .   .   .   .   .   .   .   .   .   .   .   .   .   .   .   .   .   .   .   .   .   .   .   .   .   .   .   .   .   .   .   .   .   .   .   .   74
   Lista 7 . . . . . .   .   .   .   .   .   .   .   .   .   .   .   .   .   .   .   .   .   .   .   .   .   .   .   .   .   .   .   .   .   .   .   .   .   .   .   .   78
   Lista 8 . . . . . .   .   .   .   .   .   .   .   .   .   .   .   .   .   .   .   .   .   .   .   .   .   .   .   .   .   .   .   .   .   .   .   .   .   .   .   .   81
   Lista 9 . . . . . .   .   .   .   .   .   .   .   .   .   .   .   .   .   .   .   .   .   .   .   .   .   .   .   .   .   .   .   .   .   .   .   .   .   .   .   .   84
   Lista 10 . . . . .    .   .   .   .   .   .   .   .   .   .   .   .   .   .   .   .   .   .   .   .   .   .   .   .   .   .   .   .   .   .   .   .   .   .   .   .   86

Solu¸˜es do N´ 3
    co         ıvel                                                                                                                                                       88
   Lista 1 . . . . . .   .   .   .   .   .   .   .   .   .   .   .   .   .   .   .   .   .   .   .   .   .   .   .   .   .   .   .   .   .   .   .   .   .   .   .   .    88
   Lista 2 . . . . . .   .   .   .   .   .   .   .   .   .   .   .   .   .   .   .   .   .   .   .   .   .   .   .   .   .   .   .   .   .   .   .   .   .   .   .   .    91
   Lista 3 . . . . . .   .   .   .   .   .   .   .   .   .   .   .   .   .   .   .   .   .   .   .   .   .   .   .   .   .   .   .   .   .   .   .   .   .   .   .   .    93
   Lista 4 . . . . . .   .   .   .   .   .   .   .   .   .   .   .   .   .   .   .   .   .   .   .   .   .   .   .   .   .   .   .   .   .   .   .   .   .   .   .   .    96
   Lista 5 . . . . . .   .   .   .   .   .   .   .   .   .   .   .   .   .   .   .   .   .   .   .   .   .   .   .   .   .   .   .   .   .   .   .   .   .   .   .   .   100
   Lista 6 . . . . . .   .   .   .   .   .   .   .   .   .   .   .   .   .   .   .   .   .   .   .   .   .   .   .   .   .   .   .   .   .   .   .   .   .   .   .   .   103
   Lista 7 . . . . . .   .   .   .   .   .   .   .   .   .   .   .   .   .   .   .   .   .   .   .   .   .   .   .   .   .   .   .   .   .   .   .   .   .   .   .   .   106
   Lista 8 . . . . . .   .   .   .   .   .   .   .   .   .   .   .   .   .   .   .   .   .   .   .   .   .   .   .   .   .   .   .   .   .   .   .   .   .   .   .   .   110
   Lista 9 . . . . . .   .   .   .   .   .   .   .   .   .   .   .   .   .   .   .   .   .   .   .   .   .   .   .   .   .   .   .   .   .   .   .   .   .   .   .   .   114
   Lista 10 . . . . .    .   .   .   .   .   .   .   .   .   .   .   .   .   .   .   .   .   .   .   .   .   .   .   .   .   .   .   .   .   .   .   .   .   .   .   .   118

Solu¸˜es dos Desafios
    co                                                                                                                                                                   120




iv                                                           OBMEP 2009
5




Lista 1                                     N´ 1
                                             ıvel
                                                                                          9




                                       N´ 1
                                        ıvel

Lista 1
   1. Encontro de amigos – Embora eu esteja certo de que meu rel´gio est´ adiantado 5
                                                                      o       a
      minutos, ele est´, na realidade, com 10 minutos de atraso. Por outro lado, o rel´gio do
                       a                                                              o
      meu amigo est´ realmente 5 minutos adiantado, embora ele pense que est´ correto. N´s
                     a                                                        a            o
      marcamos um encontro `s 10 horas e planejamos chegar pontualmente. Quem chegar´
                               a                                                            a
      em primeiro lugar? Depois de quanto tempo chegar´ o outro?
                                                         a

   2. Trabalho comunit´rio – Uma classe tem 22 alunos e 18 alunas. Durante as f´rias, 60%
                         a                                                       e
      dos alunos dessa classe foram prestar trabalho comunit´rio. No m´
                                                            a         ınimo, quantas alunas
      participaram desse trabalho?

                         (A) 1      (B) 2     (C) 4       (D) 6     (E) 8

      ´
   3. Area de trap´zios – Unindo quatro trap´zios
                       e                          e
      iguais de bases 30 cm e 50 cm e lados n˜o pa-
                                                a
      ralelos iguais, como o da figura, podemos formar
      um quadrado de ´rea 2 500 cm2 , com um “buraco”
                         a
      quadrado no meio. Qual ´ a ´rea de cada trap´zio,
                               e a                e
      em cm2 ?

                   (A) 200       (B) 250     (C) 300      (D) 350      (E) 400

   4. Adivinha¸˜o – Pensei em 2 n´meros de dois algarismos, que n˜o possuem algarismos
               ca                   u                              a
      em comum, sendo um o dobro do outro. Al´m disso, os algarismos do menor n´mero
                                                 e                                 u
      s˜o a soma e a diferen¸a dos algarismos do maior n´mero. Quais s˜o os n´meros?
       a                    c                           u             a      u

   5. 18 n´meros consecutivos – Escreva 18 n´meros consecutivos de 3 algarismos e veri-
          u                                  u
      fique que um deles ´ divis´ pela soma de seus algarismos.
                        e      ıvel
      Isso ´ sempre verdade. Ou seja: se vocˆ escrever 18 n´meros consecutivos de 3 algaris-
           e                                e              u
      mos, ent˜o um deles ´ divis´ pela soma de seus algarismos. Mostre este fato.
               a           e     ıvel




                                       OBMEP 2009                                             1
5




                                           N´ 1
                                            ıvel                                                                  Lista 2
           9




Lista 2
    1. Completar uma tabela – Descubra a regra utilizada para as casas j´ preenchidas e
                                                                        a
       complete a tabela. Qual ´ o valor de A?
                               e

                                       0    1    2   3     4
                                       1    2    5   10
                                       2
                                       3
                                       4                   A

    2. Procurando m´ltiplos de 9 – Consideremos um conjunto formado por 10 n´meros
                       u                                                               u
       naturais diferentes. Se calculamos todas as diferen¸as entre esses n´meros, pelo menos
                                                          c                u
       uma dessas diferen¸as ´ um m´ltiplo de 9?
                           c e        u
    3. Correndo numa pra¸a – Um atleta costuma correr
                             c
       15, 5 km ao redor de uma pra¸a retangular de dimens˜es
                                   c                      o
       900 m × 600 m. Ele inicia a corrida sempre do ponto P
       situado a 550 m de um dos v´rtices correndo no sentido
                                   e
       hor´rio, como mostra a figura. Em que ponto da pra¸a
           a                                               c
       ele para?
    4. Ovos para um bolo – Uma doceira foi ao mercado comprar ovos para fazer 43 bolos,
       todos com a mesma receita, que gasta menos de 9 ovos. O vendedor repara que se
       tentar embrulhar os ovos que a doceira comprou em grupos de 2 ou de 3 ou de 4 ou de
       5 ou de 6 ovos, sempre sobra 1 ovo. Quantos ovos ela usa em cada bolo? Qual o menor
       n´mero de ovos que a doceira vai gastar para fazer os 43 bolos?
        u
    5. C´lculos H e V – Vocˆ consegue colocar os n´meros de 1 a 8
        a                        e                      u                                 m÷ m m
                                                                                              =
       dentro dos c´ ırculos, sem repeti-los, de modo que os c´lculos na
                                                              a            −                                          ×
       horizontal e na vertical sejam corretos?                                           m                                          m
                                                                           ....................................    ....................................
       Dica: Quais as possibilidades para a multiplica¸˜o? Quais os
                                                          ca
                                                                                          m+ m m
                                                                                              =
       poss´ıveis lugares para o n´mero 1?
                                   u




2                                       OBMEP 2009
5




Lista 3                                                                                                                                        N´ 1
                                                                                                                                                ıvel
                                                                                                                                                                                                                                                                                                                                                            9




Lista 3
   1. Cortando uma cartolina – Uma folha retangular de cartolina foi cortada ao longo de
      sua diagonal. Num dos peda¸os obtidos, foram feitos 2 cortes paralelos aos 2 lados
                                   c
      menores e pelos pontos m´dios desses lados. Ao final sobrou um retˆngulo de per´
                              e                                        a            ımetro
      129 cm. O desenho abaixo indica a sequˆncia de cortes.
                                              e

                      ......................................................................................................................
                      .
                      .                                                                                                                    .
                                                                                                                                           .                                                                                                                      ........
                                                                                                                                                                                                                                                                         .
                                                                                                                                                                                                                                                                         .
                      .
                      .                                                                                                                    .
                                                                                                                                           .
                                                                                                                                           .                                                                                                               ........ .    .
                                                                                                                                                                                                                                                                         .
                      .
                      .
                      .                                                                                                                    .
                                                                                                                                           .                                                                                                        ........             .
                                                                                                                                                                                                                                                                         .
                      .
                      .
                      .                                                                                                                    .
                                                                                                                                           .
                                                                                                                                           .                                                                                                 ........                    .
                                                                                                                                                                                                                                                                         .
                                                                                                                                                                                                                                                                         .
                      .
                      .                                                                                                                    .
                                                                                                                                           .                                                                                          ........                           .
                                                                                                                                                                                                                                                                         .
                                                                                                                                                                                                                                                                         .       ............................................................
                                                                                                                                                                                                                                                                                 .                                                          .
                      .
                      .
                      .
                      .
                                                                                                                                           .
                                                                                                                                           .
                                                                                                                                           .
                                                                                                                                           .                                                                               ............                                  .
                                                                                                                                                                                                                                                                         .
                                                                                                                                                                                                                                                                         .
                                                                                                                                                                                                                                                                         .
                                                                                                                                                                                                                                                                                 .
                                                                                                                                                                                                                                                                                 .
                                                                                                                                                                                                                                                                                 .
                                                                                                                                                                                                                                                                                 .
                                                                                                                                                                                                                                                                                                                                            .
                                                                                                                                                                                                                                                                                                                                            .
                                                                                                                                                                                                                                                                                                                                            .
                                                                                                                                                                                                                                                                                                                                            .
                      .                                                                                                                    .                                                                        ........                                                     .                                                          .
                      .
                      .
                      .
                      .
                      .
                      .
                      .
                                                                                                                                           .
                                                                                                                                           .
                                                                                                                                           .
                                                                                                                                           .
                                                                                                                                           .
                                                                                                                                           .
                                                                                                                                           .   -                                                      ....
                                                                                                                                                                                                                 ....
                                                                                                                                                                                                          ........
                                                                                                                                                                                                                                                                         .
                                                                                                                                                                                                                                                                         .
                                                                                                                                                                                                                                                                         .
                                                                                                                                                                                                                                                                         .
                                                                                                                                                                                                                                                                         .
                                                                                                                                                                                                                                                                         .
                                                                                                                                                                                                                                                                         .   -   .
                                                                                                                                                                                                                                                                                 .
                                                                                                                                                                                                                                                                                 .
                                                                                                                                                                                                                                                                                 .
                                                                                                                                                                                                                                                                                 .
                                                                                                                                                                                                                                                                                 .
                                                                                                                                                                                                                                                                                                                                            .
                                                                                                                                                                                                                                                                                                                                            .
                                                                                                                                                                                                                                                                                                                                            .
                                                                                                                                                                                                                                                                                                                                            .
                                                                                                                                                                                                                                                                                                                                            .
                                                                                                                                                                                                                                                                                                                                            .
                                                                                                                                                                                                                                                                                                                                            .
                      .
                      .
                      .
                      .
                                                                                                                                           .
                                                                                                                                           .
                                                                                                                                           .
                                                                                                                                           .                                                   ........
                                                                                                                                                                                            ....
                                                                                                                                                                                                                                                                         .
                                                                                                                                                                                                                                                                         .
                                                                                                                                                                                                                                                                         .
                                                                                                                                                                                                                                                                         .
                                                                                                                                                                                                                                                                                 .
                                                                                                                                                                                                                                                                                 .
                                                                                                                                                                                                                                                                                 .
                                                                                                                                                                                                                                                                                 .
                                                                                                                                                                                                                                                                                                                                            .
                                                                                                                                                                                                                                                                                                                                            .
                                                                                                                                                                                                                                                                                                                                            .
                                                                                                                                                                                                                                                                                                                                            .
                      .
                      .
                      .
                                                                                                                                           .
                                                                                                                                           .
                                                                                                                                           .                                     ............                                                                            .
                                                                                                                                                                                                                                                                         .
                                                                                                                                                                                                                                                                         .
                                                                                                                                                                                                                                                                         .
                                                                                                                                                                                                                                                                                 .
                                                                                                                                                                                                                                                                                 .
                                                                                                                                                                                                                                                                                 .
                                                                                                                                                                                                                                                                                 .
                                                                                                                                                                                                                                                                                                                                            .
                                                                                                                                                                                                                                                                                                                                            .
                                                                                                                                                                                                                                                                                                                                            .
                                                                                                                                                                                                                                                                                                                                            .
                      .
                      .
                      .
                      .
                                                                                                                                           .
                                                                                                                                           .
                                                                                                                                           .
                                                                                                                                           .                          ............                                                                                       .
                                                                                                                                                                                                                                                                         .
                                                                                                                                                                                                                                                                         .
                                                                                                                                                                                                                                                                         .
                                                                                                                                                                                                                                                                                 ............................................................
                      .
                      .
                      .
                      .
                                                                                                                                           .
                                                                                                                                           .
                                                                                                                                           .
                                                                                                                                           .                   ........
                                                                                                                                                             ...
                                                                                                                                                                                                                                                                         .
                                                                                                                                                                                                                                                                         .
                                                                                                                                                                                                                                                                         .
                                                                                                                                                                                                                                                                         .
                      .
                      .                                                                                                                    .
                                                                                                                                           .
                      ......................................................................................................................
                      .                                                                                                                    .       ..
                                                                                                                                               .............
                                                                                                                                                    ...................................................................................................................  .
                                                                                                                                                                                                                                                                         .
                                                                                                                                                                                                                                                                         .




      Qual era o per´
                    ımetro da folha antes do corte?
   2. A soma errada – A soma ao lado est´ incorreta. Para corrigi-la basta
                                              a                                                                                                                                                                                                                                                                                                       742586
      substituir um certo algarismo em todos os lugares que ele aparece na conta                                                                                                                                                                                                                                                                     +829430
      por um outro algarismo. Quais s˜o esses dois algarismos?
                                      a                                                                                                                                                                                                                                                                                                              1212016
   3. N´mero de 5 algarismos – Os algarismos 1, 2, 3, 4 e 5 foram usados, cada um uma
        u
      unica vez, para escrever um n´mero de 5 algarismos a b c d e, tal que: a b c ´ divis´
      ´                                u                                           e      ıvel
      por 4, b c d por 5, e c d e por 3. Encontre esse n´mero.
                                                        u
   4. Tabela misteriosa – Complete a tabela 6 × 6 de modo
                                                                                                                                                                                                                                                                             32                                                                 40
      que em cada linha e cada coluna apare¸am apenas
                                              c                                                                                                                                                                                                                                                                                                      49
      m´ltiplos de um dos n´meros:
        u                  u                                                                                                                                                                                                                                                                                                           22
                                                                                                                                                                                                                                                                                                       15
                  2, 3, 4, 5, 6, 7, 8, 9, 10, 11, 12 .                                                                                                                                                                                                                                                                                 24
                                                                                                                                                                                                                                                                                                                                                     42
      Vocˆ pode repetir apenas um n´mero na tabela.
         e                         u
   5. Habitantes e esporte – Numa cidade com quase 30 mil habitantes, dois nonos dos
      homens e dois quinze avos das mulheres pratica esporte somente nos finais de semana, e
      o n´mero de habitantes que n˜o pratica esporte ´ o qu´
         u                         a                  e     ıntuplo dos que praticam esporte
      regularmente. Com esses dados, complete a tabela.


          N˜o praticam esporte
           a                                                                                     Praticam esporte somente                                                                                                                                     Praticam esporte                                                                  Popula¸˜o
                                                                                                                                                                                                                                                                                                                                                      ca
                                                                                                    nos finais de semana                                                                                                                                         regularmente
          fem.        masc.                                                                      fem.          masc.                                                                                                                                          fem.     masc.                                                                      total
          8 563       8 322                                                                                                                                                                                                                                            1 252




                                                                                                                                           OBMEP 2009                                                                                                                                                                                                           3
5




                                                N´ 1
                                                 ıvel                                                  Lista 4
             9




Lista 4
                                              PSfrag replacements
    1. Bot˜es luminosos – No mecanismo luminoso da figura,
            o
                                                                1                                 1
       cada um dos oito bot˜es pode acender as cores verde ou
                             o
                                                                2                          8            2
       azul. O mecanismo funciona do seguinte modo: ao ser
                                                                3
       ligado, todos os bot˜es acendem a luz azul, e se aperta-
                           o
                                                                4                                             3
       mos um bot˜o, esse bot˜o e seus vizinhos trocam de cor.
                   a           a                                                       7
                                                                5
       Se ligarmos o mecanismo e apertarmos sucessivamente os
                                                                6
       bot˜es 1, 3 e 5, qual ser´ o n´mero de luzes verdes que
           o                    a    u                                                     6            4
                                                                7
       estar˜o acesas no final?
             a                                                                                    5
                                                                                   8
                 (A) 3   (B) 4    (C) 5         (D) 6         (E) 7

    2. Qual ´ o n´mero? – Um n´mero de 6 algarismos come¸a por 1. Se deslocamos esse
              e    u                u                            c
       algarismo 1 da primeira posi¸˜o para a ultima ` direita, obtemos um novo n´mero de 6
                                   ca         ´      a                           u
       algarismos que ´ o triplo do n´mero de partida. Qual ´ esse n´mero?
                      e              u                        e      u

    3. Jardim variado – Um jardim retangular de 120 m por 80 m foi dividido em 6 regi˜es   o
       como na figura, onde N, M e P s˜o pontos m´dios dos lados, e R divide o comprimento
                                       a           e
       na raz˜o 1/3. Em cada regi˜o ser´ plantado um dos seguintes tipos de flor: rosa,
             a                      a      a
       margarida, cravo, bem-me-quer, violeta e brom´lia, cujos pre¸os, por m2 est˜o indicados
                                                    e              c              a
       na tabela. Quais as poss´
                               ıveis escolhas das flores em cada regi˜o, de modo a gastar o
                                                                      a
       m´ınimo poss´
                   ıvel?
                                                                             Tipo              Pre¸o por m2
                                                                                                  c
                                                                       rosa                         3,50
                                                                       margarida                    1,20
                                                                       cravo                        2,20
                                                                       bem-me-quer                  0,80
                                                                       violeta                      1,70
                                                                       brom´lia
                                                                             e                      3,00

    4. O algarismo 3 – Luis escreveu a sequˆncia de n´meros naturais a partir de 1:
                                           e         u

                                  1, 2, 3, 4, 5, 6, 7, 8, 9, 10, 11, 12, · · · .

         Quando ele escreveu o algarismo 3 pela 25a vez?

    5. Soma de potˆncias – O n´mero 3444 + 4333 ´ divis´ por 5?
                  e           u                 e      ıvel




4                                          OBMEP 2009
5




Lista 5                                         N´ 1
                                                 ıvel
                                                                                                                                                             9




Lista 5
   1. Telefonemas – Jo˜o mora em Salvador e seus pais em Recife. Para matar a saudade,
                          a
      ele telefona para seus pais a cada trˆs dias. O primeiro telefonema foi feito no domingo,
                                           e
      o segundo telefonema na 4a feira, o terceiro telefonema no s´bado, e assim por diante.
                                                                     a
      Em qual dia da semana Jo˜o telefonou para seus pais pela cent´sima vez?
                                  a                                    e

   2. O maior produto – Com os algarismos de 1 a 5 e um sinal                  X                    1                                2
      de multiplica¸˜o × Clara forma o produto de 2 n´meros, com
                   ca                                 u
      o sinal × entre eles. Como Clara deve colocar os cart˜es para
                                                           o
                                                                               3                    4                                 5
      obter o maior produto poss´ıvel?
                                                                           P
   3. O caminho da Joaninha – Dona Joaninha quer atraves-
      sar um p´tio com azulejos quadrados numerados como
                a
      mostra a figura. Ela vai partir do ponto P e quer chegar
      ao ponto C andando somente sobre os lados dos azule-
      jos. Dona Joaninha n˜o quer ter n´meros primos ` sua
                           a             u              a
      direita ao longo de todo o percurso. Qual ´ o menor
                                                   e
      percurso que ela pode fazer?                                                                                                                                  C
                                                                                                          ................................
                                                                                                 ........                                 .....
                                                                                            .....
   4. O lugar dos amigos – Sete amigos tra¸aram um triˆngulo, um
                                             c            a                          ..
                                                                                      .
                                                                                        ....                          .
                                                                                                                      .
                                                                                                                      .
                                                                                                                                               ...
                                                                                                                     ...................................................
                                                                                                                                                  ...
                                                                                                                                                     ..
                                                                                                                                                      ..
                                                                                                                                                                        .
                                                                                                                                                                        .
                                                                                                                                                                        .

      quadrado e um c´
                     ırculo. Cada um marcou seu lugar com um n´mero:
                                                              u
                                                                                   ..
                                                                                   .
                                                                                   .
                                                                                   .              1                   .
                                                                                                                      .
                                                                                                                      .
                                                                                                                      . ...
                                                                                                                      .
                                                                                                                      .
                                                                                                                      .
                                                                                                                      . .... .....
                                                                                                                                   2                   .
                                                                                                                                                       .
                                                                                                                                                       .
                                                                                                                                                                        .
                                                                                                                                                                        .
                                                                                                                                                                        .
                                                                                                                                                                        .
                                                                                                                                                                        .
                                                                                                                                                                        .
                                                                                                                                                                        .
                                                                                   .
                                                                                   .
                                                                                   ..
                                                                                    ..
                                                                                                                 .....
                                                                                                                      .
                                                                                                                      ... ....
                                                                                                                      .
                                                                                                                      .
                                                                                                                      .                  .... ..      .
                                                                                                                                                       .
                                                                                                                                                       .
                                                                                                                                                       .   7            .
                                                                                                                                                                        .
                                                                                                                                                                        .
                                                                                                                                                                        .
                                                                                                                                                                        .
                                                                                                                                                                        .
                                                                                      ...
                                                                                         .... ...... .
                                                                                                                      .
                                                                                                                      .
                                                                                                                      .
                                                                                                                      .
                                                                                                                      .     3                ........
                                                                                                                                                 .
                                                                                                                                                                        .
                                                                                                                                                                        .
                                                                                                                                                                        .
                                                                                                                                                                        .
                                                                                                                                                                        .
                                                                                                                                                                        .
                                                                                                                                                                        .
                                                                                                 .
                                                                                                     4
                                                                                             ..... ... .
                                                                                                      .
                                                                                                                      .
                                                                                                                      .
                                                                                                  .......... . ..........
                                                                                                                      .
                                                                                                                      .
                                                                                                  .. ...................                6
                                                                                                                                           ..... ......
                                                                                                                                                       ....
                                                                                                                                                                        .
                                                                                                                                                                        .
                                                                                                                                                                        .
                                                                                                                                                                        .
                                                                                                                                                                        .
                                                                                                                                                                        .
                                                                                                                                                                        .
          Ana:         “Eu   n˜o falarei nada.”
                              a                                                         ....
                                                                                            . ....
                                                                                                    5
                                                                                                                      .
                                                                                                                      .
                                                                                                                                                               ....
                                                                                                                                                                        .
                                                                                                                                                                        .
                                                                                                                     ....................................................
                                                                                                                                                            ....
                                                                                    ............................................................................
                                                                                      ..
          Bento:       “Eu   estou dentro de uma unica figura.”
                                                   ´
          Celina:      “Eu   estou dentro das trˆs figuras.”
                                                 e
          Diana:       “Eu   estou dentro do triˆngulo mas n˜o do quadrado.”
                                                 a           a
          Elisa:       “Eu   estou dentro do triˆngulo e do c´
                                                 a           ırculo.”
          F´bio:
           a           “Eu   n˜o estou dentro de um pol´
                              a                          ıgono.”
          Guilherme:   “Eu   estou dentro do c´ırculo.”

      Encontre o lugar de cada um.

   5. Quadrado perfeito? – Cada um dos cinco n´meros abaixo tem 100 algarismos, e ´
                                                   u                              e
      formado pela repeti¸˜o de um ou dois algarismos:
                         ca

                                             N1 = 333333 . . . 3
                                             N2 = 666666 . . . 6
                                             N3 = 151515 . . . 15
                                             N4 = 212121 . . . 21
                                             N5 = 272727 . . . 27

      Algum destes n´meros ´ um quadrado perfeito?
                    u      e




                                             OBMEP 2009                                                                                                             5
5




                                               N´ 1
                                                ıvel                                      Lista 6
             9




Lista 6
    1. Preenchendo quadradinhos – Complete os quadradinhos com os n´meros 1, 2, 3, 5, 6.
                                                                   u

                                         +     −        ×   ÷     = 4



    2. Os 3 n´meros – Sofia brinca de escrever todos os n´meros de 4 algarismos diferentes
               u                                            u
       que se pode escrever com os algarismos 1, 2, 4 e 7. Ela soma 3 desses n´meros – todos
                                                                              u
       diferentes – e obt´m 13 983. Quais s˜o esses 3 n´meros?
                         e                 a            u
    3. Preencher uma tabela – Jandira deve preencher uma tabela
       4 × 4 que j´ vem com duas casas preenchidas com os n´meros
                   a                                         u
       1 e 2 - veja ao lado. Duas casas s˜o consideradas vizinhas se
                                         a
                                                                                  1   2
       tˆm um v´rtice ou um lado em comum.
        e        e
       As regras que ela tem que obedecer s˜o:
                                           a

           • uma casa s´ pode ser preenchida se alguma de suas casas vizinhas j´ cont´m um
                       o                                                       a     e
             n´mero;
              u
           • ao preencher uma casa, deve-se colocar a soma de todos os n´meros que j´ constam
                                                                        u           a
             em suas casas vizinhas.

         Qual ´ o maior n´mero que ´ poss´ escrever na tabela?
              e          u         e     ıvel

    4. Olimp´ ıada de Pequim – Na Olimp´  ıada de Pequim sentaram-se, em uma mesa quadrada,
       as mulheres, Maria e Tˆnia, e os homens, Juan e David, todos atletas. Cada um deles
                               a
       pratica um esporte diferente: nata¸˜o, vˆlei, gin´stica e atletismo. Eles estavam sentados
                                         ca o           a
       da seguinte maneira:

          (a) Quem pratica a nata¸˜o estava ` esquerda de Maria.
                                 ca         a
         (b) Quem pratica gin´stica estava em frente a Juan.
                             a
          (c) Tˆnia e David sentaram-se lado a lado.
               a
         (d) Uma mulher sentou-se ao lado de quem pratica volei.

         Qual dos atletas pratica atletismo?

    5. Culturas diferentes – Jorge, que mora em Recife, se corresponde com seu amigo inglˆs
                                                                                          e
       Ralph que mora na Inglaterra. Os dois se compreendem muito bem nas duas l´    ınguas,
       mas tˆm um problema com as datas: a data 08/10 no Brasil significa 8 de outubro, e na
             e
       Inglaterra 10 de agosto. Por causa disso, os dois combinaram n˜o se escrever nos dias
                                                                     a
       em que a data for amb´ ıgua. Eles preferem datas como 25/03 que s´ pode significar 25
                                                                        o
       de mar¸o.
               c

          (a) Em quais das datas a seguir Jorge e Ralph n˜o podem se escrever?
                                                         a
                       (i) 3 de dezembro           (ii) 18 de agosto    (iii) 5 de maio
         (b) Quando ocorre o maior per´
                                      ıodo em que os dois amigos n˜o podem se escrever?
                                                                  a


6                                          OBMEP 2009
5




Lista 7                                     N´ 1
                                             ıvel
                                                                                          9




Lista 7
   1. Uma liquida¸˜o – Na liquida¸˜o da loja SUPER-SUPER todos os produtos est˜o 50%
                  ca               ca                                              a
      mais baratos, e aos s´bados existe ainda um desconto adicional de 20%. Carla com-
                           a
      prou uma cal¸a antes da liquida¸˜o, e agora ela se lamenta: Nesse s´bado eu teria
                   c                  ca                                    a
      economizado R$ 50,40 na cal¸a. Qual era o pre¸o da cal¸a antes da liquida¸˜o?
                                  c                c        c                  ca

   2. N´mero com muitos zeros – Se a ´ o n´mero 0, 000 . . . 000 1, ent˜o qual das express˜es
       u                             e u                               a                  o
                                                          2009 zeros
      a seguir representa o maior n´mero?
                                   u

           (A)    3+a        (B)   3−a        (C)    3a         (D)    3/a   (E)   a/3

   3. Corrida das tartarugas – Cinco tartarugas apostaram uma corrida em linha reta e na
      chegada a situa¸˜o foi a seguinte: Sininha est´ 10 m atr´s de Olguinha e 25 m ` frente
                     ca                             a         a                     a
      de Rosinha que est´ 5 m atr´s de Elzinha que est´ 25 m atr´s de Pulinha. Qual foi a
                        a          a                     a         a
      ordem de chegada?

   4. Que mem´ria... – Esquecinaldo tem p´ssima mem´ria para guardar n´meros, mas
                  o                            e            o                 u
      o
      ´tima para lembrar sequˆncias de opera¸˜es. Por isso, para lembrar do seu c´digo
                                e              co                                     o
      banc´rio de 5 algarismos, ele consegue se lembrar que nenhum dos algarismos ´ zero,
            a                                                                       e
      os dois primeiros algarismos formam uma potˆncia de 5, os dois ultimos formam uma
                                                    e                 ´
      potˆncia de 2, o do meio ´ um m´ltiplo de 3 e a soma de todos os algarismos ´ um
          e                       e      u                                            e
      n´mero ´
        u       ımpar. Agora ele n˜o precisa mais decorar o n´mero porque ele sabe que ´ o
                                   a                          u                         e
      maior n´mero que satisfaz essas condi¸˜es e que n˜o tem algarismos repetidos. Qual ´
               u                            co          a                                 e
      esse c´digo?
             o

   5. Uma fra¸˜o irredut´
                ca          ıvel – Encontre uma fra¸˜o irredut´ tal que o produto de seu
                                                   ca         ıvel
      numerador pelo denominador seja 2 × 3 × 4 × 5 × . . . × 10. Quantas dessas fra¸˜es
                                                                                    co
      irredut´
             ıveis existem?




                                       OBMEP 2009                                             7
5




                                                N´ 1
                                                 ıvel                                                Lista 8
             9




Lista 8
    1. Transformar em decimal – Escreva o resultado das seguintes express˜es na forma
                                                                         o
       decimal:
                            2             5                      5                             2
                 (a)   7×       + 16 ×         (b)    5− 2÷                (c)    1+            3
                            3            12                      3
                                                                                          1+
                                                                                               1+4

    2. Uma sequˆncia especial – Escrevendo sucessivamente os n´meros naturais, obtemos
                  e                                           u
       a sequˆncia:
             e

                         1 2 3 4 5 6 7 8 9 10 11 12 13 14 15 16 17 18 19 20 21 22 . . .

         Qual algarismo est´ na 2 009a posi¸˜o dessa sequˆncia?
                           a               ca            e

    3. Cortar um retˆngulo – Como cortar um retˆngulo de 13 cm por 7 cm em 13 retˆngulos
                    a                          a                                 a
       diferentes?

    4. Medida de ˆngulo – Na figura, AOD e B OY s˜o ˆngulos retos e a medida de DOY
                    a                                   a a
       est´ entre 40◦ e 50◦ . Al´m disso, os pontos C e Y est˜o sobre a reta r, enquanto
          a                     e                                a
       D e E est˜o sobre a reta s. Os poss´
                  a                         ıveis valores para a medida de AOC variam de:

                                                                 (A)   30◦ a 40◦
                                                                 (B)   40◦ a 50◦
                                                                 (C)   50◦ a 60◦
                                                                 (D)   40◦ a 60◦
                                                                 (E)   n˜o podem ser determinados
                                                                        a




                                              √
    5. Per´ımetros e ´reas – Um quadrado tem 3 √ 3 cm de lado, e as dimens˜es de um
                     a               √      √   +                         o
                         ımetros, s˜o 72 + 3 6 e 2. Qual dos dois tem maior ´rea? E
       retˆngulo, em cent´
          a                        a                                        a
       maior per´
                ımetro?


    6. C´lculo de ˆngulo – Encontre B AD, sabendo
        a         a
       que DAC = 39◦ , AB = AC e AD = BD.




8                                             OBMEP 2009
5




Lista 9                                    N´ 1
                                            ıvel
                                                                                          9




Lista 9
   1. O caminho da formiga – Uma formiga sai de um ponto A, anda 7 cm para a esquerda,
      5 cm para cima, 3 cm para a direita, 2 cm para baixo, 9 cm para a direita, 2 cm para
      baixo, 1 cm para a esquerda e 1 cm para baixo, chegando no ponto B. Qual ´ a distˆncia
                                                                               e       a
      d entre A e B?

               (A) 0 cm      (B) 1 cm       (C) 4 cm       (D) 5 cm       (E) 7 cm

   2. Menino mentiroso – Jo˜ozinho mente nas ter¸as-feiras, quintas-feiras e s´bados e o
                                 a                     c                         a
      resto dos dias fala a verdade. Um dia Pedrinho encontra com Jo˜ozinho e tˆm o seguinte
                                                                    a          e
      di´logo:
        a

          • Pedrinho pergunta: Que dia ´ hoje?
                                       e
          • Jo˜ozinho responde: S´bado.
              a                  a
          • Pedrinho pergunta: E que dia ser´ amanh˜?
                                            a      a
          • Jo˜ozinho responde: Quarta-feira.
              a

      Que dia da semana o Pedrinho encontrou com o Jo˜ozinho?
                                                     a

   3. Encontre os 4 n´meros – Encontre quatro n´meros distintos de 3 algarismos, tais que
                      u                           u
      a soma de trˆs quaisquer deles ´ divis´ pelo quarto n´mero.
                  e                  e      ıvel           u
   4. Colando 6 triˆngulos – Construa 6 triˆngulos
                        a                             a
      equil´teros, o primeiro com lado de comprimento 1 cm e
           a
      os triˆngulos seguintes com lado igual a metade do lado
            a
      do triˆngulo anterior, como indicado na figura ao lado.
            a
      Qual ´ o per´
            e      ımetro desta figura?

   5. Os livros da Elisa – Elisa tem 24 livros de ciˆncias e outros de matem´tica e litera-
                                                     e                         a
                                                                   1
      tura. Se Elisa tivesse um livro a mais de matem´tica, ent˜o de seus livros seria de
                                                       a       a
                                                                   9
      matem´tica e um quarto de literatura. Se Elisa tem menos que 100 livros, quantos livros
             a
      de matem´tica ela possui?
               a




                                       OBMEP 2009                                             9
5




                                                   N´ 1
                                                    ıvel                                Lista 10
              9




Lista 10
     1. Divis˜o por 9 –
             a

           (a) Listemos os primeiros 20 092 009 n´meros naturais. Em seguida, substitu´
                                                 u                                     ımos,
               sucessivamente, cada n´mero pela soma dos seus algarismos, at´ obtermos uma
                                      u                                      e
               lista de n´meros com apenas um algarismo. A lista tem mais algarismos 4 ou 5?
                         u
               Quantos 9 tem a lista?
          (b) Aplicando o mesmo processo ao n´mero 32 009 , isto ´, substituindo o n´mero pela
                                               u                 e                  u
              soma dos seus algarismos, qual ´ o n´mero de apenas um algarismo obtido?
                                             e    u
           (c) E para o n´mero 172 009 ?
                         u

     2. Uma brincadeira na sala de aula – A professora Raquel inventou a seguinte brincadeira:
        escreva um n´mero no quadro, se ele for ´
                       u                        ımpar acrescente 3 unidades ao n´mero, e se
                                                                                  u
        ele for par divida o n´mero por 2.
                              u
          Esta opera¸˜o pode ser feita diversas vezes. A professora est´ interessada em obter no
                    ca                                                 a
          final o n´mero 1 e perguntou para a classe: Como obter o n´mero 1 ap´s 3 opera¸˜es?
                  u                                                   u           o        co
          E ap´s 4 opera¸˜es? E ap´s 5 opera¸˜es?
              o         co         o          co

     3. Calcule a idade – Laura e sua av´ Ana acabaram de descobrir que, no ano passado,
                                           o
        suas idades eram divis´
                              ıveis por 8 e, no pr´ximo ano, ser˜o divis´
                                                  o             a       ıveis por 7. Vov´ Ana
                                                                                        o
        ainda n˜o ´ centen´ria. Qual ´ a idade de Laura?
               a e        a           e

     4. Divis˜es e restos – O dobro de um n´mero dividido por 5 deixa resto 1. Qual o resto
             o                             u
        da divis˜o desse n´mero por 5?
                a         u

     5. Preenchendo o c´ ırculo – Cada um dos sinais , , , e representa um n´mero   u
        de 1 algarismo. Descubra quem s˜o eles e complete o n´mero que falta no c´
                                       a                     u                   ırculo em
        branco.
                                                                   
                             ×             ×   /           ×        +
                         47 − − 423 − − − 282 − −
                            −→       − −→     −→                   − − → 1448
                                                                    −−
                                                                   




10                                         OBMEP 2009
Uma palavra aos alunos e professores sobre o Banco de Questões de Matemática
Uma palavra aos alunos e professores sobre o Banco de Questões de Matemática
Uma palavra aos alunos e professores sobre o Banco de Questões de Matemática
Uma palavra aos alunos e professores sobre o Banco de Questões de Matemática
Uma palavra aos alunos e professores sobre o Banco de Questões de Matemática
Uma palavra aos alunos e professores sobre o Banco de Questões de Matemática
Uma palavra aos alunos e professores sobre o Banco de Questões de Matemática
Uma palavra aos alunos e professores sobre o Banco de Questões de Matemática
Uma palavra aos alunos e professores sobre o Banco de Questões de Matemática
Uma palavra aos alunos e professores sobre o Banco de Questões de Matemática
Uma palavra aos alunos e professores sobre o Banco de Questões de Matemática
Uma palavra aos alunos e professores sobre o Banco de Questões de Matemática
Uma palavra aos alunos e professores sobre o Banco de Questões de Matemática
Uma palavra aos alunos e professores sobre o Banco de Questões de Matemática
Uma palavra aos alunos e professores sobre o Banco de Questões de Matemática
Uma palavra aos alunos e professores sobre o Banco de Questões de Matemática
Uma palavra aos alunos e professores sobre o Banco de Questões de Matemática
Uma palavra aos alunos e professores sobre o Banco de Questões de Matemática
Uma palavra aos alunos e professores sobre o Banco de Questões de Matemática
Uma palavra aos alunos e professores sobre o Banco de Questões de Matemática
Uma palavra aos alunos e professores sobre o Banco de Questões de Matemática
Uma palavra aos alunos e professores sobre o Banco de Questões de Matemática
Uma palavra aos alunos e professores sobre o Banco de Questões de Matemática
Uma palavra aos alunos e professores sobre o Banco de Questões de Matemática
Uma palavra aos alunos e professores sobre o Banco de Questões de Matemática
Uma palavra aos alunos e professores sobre o Banco de Questões de Matemática
Uma palavra aos alunos e professores sobre o Banco de Questões de Matemática
Uma palavra aos alunos e professores sobre o Banco de Questões de Matemática
Uma palavra aos alunos e professores sobre o Banco de Questões de Matemática
Uma palavra aos alunos e professores sobre o Banco de Questões de Matemática
Uma palavra aos alunos e professores sobre o Banco de Questões de Matemática
Uma palavra aos alunos e professores sobre o Banco de Questões de Matemática
Uma palavra aos alunos e professores sobre o Banco de Questões de Matemática
Uma palavra aos alunos e professores sobre o Banco de Questões de Matemática
Uma palavra aos alunos e professores sobre o Banco de Questões de Matemática
Uma palavra aos alunos e professores sobre o Banco de Questões de Matemática
Uma palavra aos alunos e professores sobre o Banco de Questões de Matemática
Uma palavra aos alunos e professores sobre o Banco de Questões de Matemática
Uma palavra aos alunos e professores sobre o Banco de Questões de Matemática
Uma palavra aos alunos e professores sobre o Banco de Questões de Matemática
Uma palavra aos alunos e professores sobre o Banco de Questões de Matemática
Uma palavra aos alunos e professores sobre o Banco de Questões de Matemática
Uma palavra aos alunos e professores sobre o Banco de Questões de Matemática
Uma palavra aos alunos e professores sobre o Banco de Questões de Matemática
Uma palavra aos alunos e professores sobre o Banco de Questões de Matemática
Uma palavra aos alunos e professores sobre o Banco de Questões de Matemática
Uma palavra aos alunos e professores sobre o Banco de Questões de Matemática
Uma palavra aos alunos e professores sobre o Banco de Questões de Matemática
Uma palavra aos alunos e professores sobre o Banco de Questões de Matemática
Uma palavra aos alunos e professores sobre o Banco de Questões de Matemática
Uma palavra aos alunos e professores sobre o Banco de Questões de Matemática
Uma palavra aos alunos e professores sobre o Banco de Questões de Matemática
Uma palavra aos alunos e professores sobre o Banco de Questões de Matemática
Uma palavra aos alunos e professores sobre o Banco de Questões de Matemática
Uma palavra aos alunos e professores sobre o Banco de Questões de Matemática
Uma palavra aos alunos e professores sobre o Banco de Questões de Matemática
Uma palavra aos alunos e professores sobre o Banco de Questões de Matemática
Uma palavra aos alunos e professores sobre o Banco de Questões de Matemática
Uma palavra aos alunos e professores sobre o Banco de Questões de Matemática
Uma palavra aos alunos e professores sobre o Banco de Questões de Matemática
Uma palavra aos alunos e professores sobre o Banco de Questões de Matemática
Uma palavra aos alunos e professores sobre o Banco de Questões de Matemática
Uma palavra aos alunos e professores sobre o Banco de Questões de Matemática
Uma palavra aos alunos e professores sobre o Banco de Questões de Matemática
Uma palavra aos alunos e professores sobre o Banco de Questões de Matemática
Uma palavra aos alunos e professores sobre o Banco de Questões de Matemática
Uma palavra aos alunos e professores sobre o Banco de Questões de Matemática
Uma palavra aos alunos e professores sobre o Banco de Questões de Matemática
Uma palavra aos alunos e professores sobre o Banco de Questões de Matemática
Uma palavra aos alunos e professores sobre o Banco de Questões de Matemática
Uma palavra aos alunos e professores sobre o Banco de Questões de Matemática
Uma palavra aos alunos e professores sobre o Banco de Questões de Matemática
Uma palavra aos alunos e professores sobre o Banco de Questões de Matemática
Uma palavra aos alunos e professores sobre o Banco de Questões de Matemática
Uma palavra aos alunos e professores sobre o Banco de Questões de Matemática
Uma palavra aos alunos e professores sobre o Banco de Questões de Matemática
Uma palavra aos alunos e professores sobre o Banco de Questões de Matemática
Uma palavra aos alunos e professores sobre o Banco de Questões de Matemática
Uma palavra aos alunos e professores sobre o Banco de Questões de Matemática
Uma palavra aos alunos e professores sobre o Banco de Questões de Matemática
Uma palavra aos alunos e professores sobre o Banco de Questões de Matemática
Uma palavra aos alunos e professores sobre o Banco de Questões de Matemática
Uma palavra aos alunos e professores sobre o Banco de Questões de Matemática
Uma palavra aos alunos e professores sobre o Banco de Questões de Matemática
Uma palavra aos alunos e professores sobre o Banco de Questões de Matemática
Uma palavra aos alunos e professores sobre o Banco de Questões de Matemática
Uma palavra aos alunos e professores sobre o Banco de Questões de Matemática
Uma palavra aos alunos e professores sobre o Banco de Questões de Matemática
Uma palavra aos alunos e professores sobre o Banco de Questões de Matemática
Uma palavra aos alunos e professores sobre o Banco de Questões de Matemática
Uma palavra aos alunos e professores sobre o Banco de Questões de Matemática
Uma palavra aos alunos e professores sobre o Banco de Questões de Matemática
Uma palavra aos alunos e professores sobre o Banco de Questões de Matemática
Uma palavra aos alunos e professores sobre o Banco de Questões de Matemática
Uma palavra aos alunos e professores sobre o Banco de Questões de Matemática
Uma palavra aos alunos e professores sobre o Banco de Questões de Matemática
Uma palavra aos alunos e professores sobre o Banco de Questões de Matemática
Uma palavra aos alunos e professores sobre o Banco de Questões de Matemática
Uma palavra aos alunos e professores sobre o Banco de Questões de Matemática
Uma palavra aos alunos e professores sobre o Banco de Questões de Matemática
Uma palavra aos alunos e professores sobre o Banco de Questões de Matemática
Uma palavra aos alunos e professores sobre o Banco de Questões de Matemática
Uma palavra aos alunos e professores sobre o Banco de Questões de Matemática
Uma palavra aos alunos e professores sobre o Banco de Questões de Matemática
Uma palavra aos alunos e professores sobre o Banco de Questões de Matemática
Uma palavra aos alunos e professores sobre o Banco de Questões de Matemática
Uma palavra aos alunos e professores sobre o Banco de Questões de Matemática
Uma palavra aos alunos e professores sobre o Banco de Questões de Matemática
Uma palavra aos alunos e professores sobre o Banco de Questões de Matemática
Uma palavra aos alunos e professores sobre o Banco de Questões de Matemática
Uma palavra aos alunos e professores sobre o Banco de Questões de Matemática
Uma palavra aos alunos e professores sobre o Banco de Questões de Matemática
Uma palavra aos alunos e professores sobre o Banco de Questões de Matemática
Uma palavra aos alunos e professores sobre o Banco de Questões de Matemática

Más contenido relacionado

Destacado

Patologia instersticial pulmonar
Patologia instersticial pulmonarPatologia instersticial pulmonar
Patologia instersticial pulmonarImagenes Haedo
 
Políticas de Desenvolvimento Regional no RGS - Antônio Paulo Cargnin
Políticas de Desenvolvimento Regional no RGS - Antônio Paulo CargninPolíticas de Desenvolvimento Regional no RGS - Antônio Paulo Cargnin
Políticas de Desenvolvimento Regional no RGS - Antônio Paulo CargninFundação de Economia e Estatística
 
4 Por Seu Imenso Amor
4   Por Seu Imenso Amor4   Por Seu Imenso Amor
4 Por Seu Imenso Amorambcarioca
 
A construção de práticas pedagógicas a partir apresentaçao
A construção de práticas pedagógicas a partir  apresentaçaoA construção de práticas pedagógicas a partir  apresentaçao
A construção de práticas pedagógicas a partir apresentaçaoLILIANE OLIVEIRA
 
Vulnerabilidad a la inseguridad alimentaria
Vulnerabilidad a la inseguridad alimentariaVulnerabilidad a la inseguridad alimentaria
Vulnerabilidad a la inseguridad alimentariaAnibal Velasquez
 
NMI13 Petr Andrýsek - Cliché a komunity na facebooku
NMI13 Petr Andrýsek - Cliché a komunity na facebookuNMI13 Petr Andrýsek - Cliché a komunity na facebooku
NMI13 Petr Andrýsek - Cliché a komunity na facebookuNew Media Inspiration
 
Eproinfobiancareginachristiane
EproinfobiancareginachristianeEproinfobiancareginachristiane
Eproinfobiancareginachristianebiancams
 
Jovens Brasileiros e o Mundo Mobile
Jovens Brasileiros e o Mundo MobileJovens Brasileiros e o Mundo Mobile
Jovens Brasileiros e o Mundo MobileElife Brasil
 
Mini Curso Banco De Dados – Unicerp
Mini Curso Banco De Dados – UnicerpMini Curso Banco De Dados – Unicerp
Mini Curso Banco De Dados – UnicerpRafael Pinheiro
 
Aula 6 Metodologia
Aula 6 MetodologiaAula 6 Metodologia
Aula 6 Metodologiagestao
 

Destacado (20)

Livro onde ficaoparaiso-2
Livro onde ficaoparaiso-2Livro onde ficaoparaiso-2
Livro onde ficaoparaiso-2
 
Web 2.0
Web 2.0Web 2.0
Web 2.0
 
Patologia instersticial pulmonar
Patologia instersticial pulmonarPatologia instersticial pulmonar
Patologia instersticial pulmonar
 
Up Side Araguaia
Up Side AraguaiaUp Side Araguaia
Up Side Araguaia
 
Políticas de Desenvolvimento Regional no RGS - Antônio Paulo Cargnin
Políticas de Desenvolvimento Regional no RGS - Antônio Paulo CargninPolíticas de Desenvolvimento Regional no RGS - Antônio Paulo Cargnin
Políticas de Desenvolvimento Regional no RGS - Antônio Paulo Cargnin
 
4 Por Seu Imenso Amor
4   Por Seu Imenso Amor4   Por Seu Imenso Amor
4 Por Seu Imenso Amor
 
Ciencia y tecn
Ciencia y tecnCiencia y tecn
Ciencia y tecn
 
A construção de práticas pedagógicas a partir apresentaçao
A construção de práticas pedagógicas a partir  apresentaçaoA construção de práticas pedagógicas a partir  apresentaçao
A construção de práticas pedagógicas a partir apresentaçao
 
Ped - junho de 2012 relativa estabilidade da taxa de desemprego
Ped - junho de 2012 relativa estabilidade da taxa de desempregoPed - junho de 2012 relativa estabilidade da taxa de desemprego
Ped - junho de 2012 relativa estabilidade da taxa de desemprego
 
Hoje (B.F)
Hoje (B.F)Hoje (B.F)
Hoje (B.F)
 
Vulnerabilidad a la inseguridad alimentaria
Vulnerabilidad a la inseguridad alimentariaVulnerabilidad a la inseguridad alimentaria
Vulnerabilidad a la inseguridad alimentaria
 
NMI13 Petr Andrýsek - Cliché a komunity na facebooku
NMI13 Petr Andrýsek - Cliché a komunity na facebookuNMI13 Petr Andrýsek - Cliché a komunity na facebooku
NMI13 Petr Andrýsek - Cliché a komunity na facebooku
 
Eproinfobiancareginachristiane
EproinfobiancareginachristianeEproinfobiancareginachristiane
Eproinfobiancareginachristiane
 
Jaqueline
JaquelineJaqueline
Jaqueline
 
Escritura s ltda
Escritura s ltdaEscritura s ltda
Escritura s ltda
 
Jovens Brasileiros e o Mundo Mobile
Jovens Brasileiros e o Mundo MobileJovens Brasileiros e o Mundo Mobile
Jovens Brasileiros e o Mundo Mobile
 
Mini Curso Banco De Dados – Unicerp
Mini Curso Banco De Dados – UnicerpMini Curso Banco De Dados – Unicerp
Mini Curso Banco De Dados – Unicerp
 
Folha 13 aula sais
Folha 13 aula saisFolha 13 aula sais
Folha 13 aula sais
 
Aula 6 Metodologia
Aula 6 MetodologiaAula 6 Metodologia
Aula 6 Metodologia
 
Grand Hyatt
Grand HyattGrand Hyatt
Grand Hyatt
 

Similar a Uma palavra aos alunos e professores sobre o Banco de Questões de Matemática

Ler e escrever diferente
Ler e escrever diferenteLer e escrever diferente
Ler e escrever diferenteEdneia Silva
 
Lp conteudo prof_3_ano
Lp conteudo prof_3_anoLp conteudo prof_3_ano
Lp conteudo prof_3_anoAmanda Bassi
 
Caderno do Gestor Vol 3 2008
Caderno do Gestor Vol 3 2008 Caderno do Gestor Vol 3 2008
Caderno do Gestor Vol 3 2008 Gelson Rocha
 
Banco de questões olimpiada de mat
Banco de questões olimpiada de matBanco de questões olimpiada de mat
Banco de questões olimpiada de matafrodite2007
 
Lp conteúdo - prof - 1ºano
Lp   conteúdo - prof - 1ºanoLp   conteúdo - prof - 1ºano
Lp conteúdo - prof - 1ºanoThais F. G. Rocha
 
Banco Questoes 2007 Da Obmep
Banco Questoes 2007 Da ObmepBanco Questoes 2007 Da Obmep
Banco Questoes 2007 Da Obmepguestbf5561
 
Normas para trabalhos acadêmicos ifpr
Normas para trabalhos acadêmicos ifprNormas para trabalhos acadêmicos ifpr
Normas para trabalhos acadêmicos ifprLuanaana
 
36529302 guia-dop-professor
36529302 guia-dop-professor36529302 guia-dop-professor
36529302 guia-dop-professorrosemereporto
 
Guia planejamento orient_did_terceira_volume_unico
Guia planejamento orient_did_terceira_volume_unicoGuia planejamento orient_did_terceira_volume_unico
Guia planejamento orient_did_terceira_volume_unicoEdson Virginio
 
36529302 guia-dop-professor
36529302 guia-dop-professor36529302 guia-dop-professor
36529302 guia-dop-professorMonica Cardoso
 
experiencias-matematicas-8-serie.pdf
experiencias-matematicas-8-serie.pdfexperiencias-matematicas-8-serie.pdf
experiencias-matematicas-8-serie.pdfchico90
 
experiencias-matematicas-8-serie.pdf
experiencias-matematicas-8-serie.pdfexperiencias-matematicas-8-serie.pdf
experiencias-matematicas-8-serie.pdfchico90
 
1bim 9anocienciasprofessor-151003012631-lva1-app6892
1bim 9anocienciasprofessor-151003012631-lva1-app68921bim 9anocienciasprofessor-151003012631-lva1-app6892
1bim 9anocienciasprofessor-151003012631-lva1-app6892Sandra Borges
 
Blog Como Ferramenta PedagóGica
Blog Como Ferramenta PedagóGicaBlog Como Ferramenta PedagóGica
Blog Como Ferramenta PedagóGicaGnomeKilla
 

Similar a Uma palavra aos alunos e professores sobre o Banco de Questões de Matemática (20)

Ler e escrever diferente
Ler e escrever diferenteLer e escrever diferente
Ler e escrever diferente
 
Lp conteudo prof_3_ano
Lp conteudo prof_3_anoLp conteudo prof_3_ano
Lp conteudo prof_3_ano
 
Caderno do Gestor Vol 3 2008
Caderno do Gestor Vol 3 2008 Caderno do Gestor Vol 3 2008
Caderno do Gestor Vol 3 2008
 
Banco de questões olimpiada de mat
Banco de questões olimpiada de matBanco de questões olimpiada de mat
Banco de questões olimpiada de mat
 
Lp conteúdo - prof - 1ºano
Lp   conteúdo - prof - 1ºanoLp   conteúdo - prof - 1ºano
Lp conteúdo - prof - 1ºano
 
Lp conteúdo - prof - 1ºano
Lp   conteúdo - prof - 1ºanoLp   conteúdo - prof - 1ºano
Lp conteúdo - prof - 1ºano
 
Banco Questoes 2007 Da Obmep
Banco Questoes 2007 Da ObmepBanco Questoes 2007 Da Obmep
Banco Questoes 2007 Da Obmep
 
Relatório
RelatórioRelatório
Relatório
 
Normas para trabalhos acadêmicos ifpr
Normas para trabalhos acadêmicos ifprNormas para trabalhos acadêmicos ifpr
Normas para trabalhos acadêmicos ifpr
 
Numeros reais
Numeros reaisNumeros reais
Numeros reais
 
Dossiê do Professor.pdf
Dossiê do Professor.pdfDossiê do Professor.pdf
Dossiê do Professor.pdf
 
36529302 guia-dop-professor
36529302 guia-dop-professor36529302 guia-dop-professor
36529302 guia-dop-professor
 
Guia planejamento orient_did_terceira_volume_unico
Guia planejamento orient_did_terceira_volume_unicoGuia planejamento orient_did_terceira_volume_unico
Guia planejamento orient_did_terceira_volume_unico
 
36529302 guia-dop-professor
36529302 guia-dop-professor36529302 guia-dop-professor
36529302 guia-dop-professor
 
experiencias-matematicas-8-serie.pdf
experiencias-matematicas-8-serie.pdfexperiencias-matematicas-8-serie.pdf
experiencias-matematicas-8-serie.pdf
 
experiencias-matematicas-8-serie.pdf
experiencias-matematicas-8-serie.pdfexperiencias-matematicas-8-serie.pdf
experiencias-matematicas-8-serie.pdf
 
1bim 9ano ciencias-professor
1bim   9ano ciencias-professor1bim   9ano ciencias-professor
1bim 9ano ciencias-professor
 
1bim 9anocienciasprofessor-151003012631-lva1-app6892
1bim 9anocienciasprofessor-151003012631-lva1-app68921bim 9anocienciasprofessor-151003012631-lva1-app6892
1bim 9anocienciasprofessor-151003012631-lva1-app6892
 
Tcc completo modelo (2013)
Tcc completo modelo (2013)Tcc completo modelo (2013)
Tcc completo modelo (2013)
 
Blog Como Ferramenta PedagóGica
Blog Como Ferramenta PedagóGicaBlog Como Ferramenta PedagóGica
Blog Como Ferramenta PedagóGica
 

Más de Antonio Carneiro (20)

Volumes 17122016
Volumes 17122016Volumes 17122016
Volumes 17122016
 
Sessão de cônicas 17122016
Sessão de cônicas 17122016Sessão de cônicas 17122016
Sessão de cônicas 17122016
 
Angulos 17122016
Angulos 17122016Angulos 17122016
Angulos 17122016
 
Estudodareta 17122016
Estudodareta 17122016Estudodareta 17122016
Estudodareta 17122016
 
Função de 2º grau 17122016
Função de 2º grau 17122016Função de 2º grau 17122016
Função de 2º grau 17122016
 
Polinomios 17122016
Polinomios 17122016Polinomios 17122016
Polinomios 17122016
 
Matrizes 17122016
Matrizes 17122016Matrizes 17122016
Matrizes 17122016
 
Introduomatemticacomercialefinanceira 17122016
Introduomatemticacomercialefinanceira 17122016Introduomatemticacomercialefinanceira 17122016
Introduomatemticacomercialefinanceira 17122016
 
Matriz
MatrizMatriz
Matriz
 
Polinomios
PolinomiosPolinomios
Polinomios
 
Matrizes
Matrizes Matrizes
Matrizes
 
Matrizes
MatrizesMatrizes
Matrizes
 
Ângulo
ÂnguloÂngulo
Ângulo
 
Função do 2º Grau.
Função do 2º Grau.Função do 2º Grau.
Função do 2º Grau.
 
Estudo da reta
Estudo da retaEstudo da reta
Estudo da reta
 
Matemática Comercial e Financeira
 Matemática Comercial e Financeira Matemática Comercial e Financeira
Matemática Comercial e Financeira
 
Sessões Cônicas
 Sessões Cônicas Sessões Cônicas
Sessões Cônicas
 
Triângulo
TriânguloTriângulo
Triângulo
 
Produtos notaveis
Produtos notaveisProdutos notaveis
Produtos notaveis
 
Função Exponencial
Função ExponencialFunção Exponencial
Função Exponencial
 

Último

LEMBRANDO A MORTE E CELEBRANDO A RESSUREIÇÃO
LEMBRANDO A MORTE E CELEBRANDO A RESSUREIÇÃOLEMBRANDO A MORTE E CELEBRANDO A RESSUREIÇÃO
LEMBRANDO A MORTE E CELEBRANDO A RESSUREIÇÃOColégio Santa Teresinha
 
HORA DO CONTO5_BECRE D. CARLOS I_2023_2024
HORA DO CONTO5_BECRE D. CARLOS I_2023_2024HORA DO CONTO5_BECRE D. CARLOS I_2023_2024
HORA DO CONTO5_BECRE D. CARLOS I_2023_2024Sandra Pratas
 
Investimentos. EDUCAÇÃO FINANCEIRA 8º ANO
Investimentos. EDUCAÇÃO FINANCEIRA 8º ANOInvestimentos. EDUCAÇÃO FINANCEIRA 8º ANO
Investimentos. EDUCAÇÃO FINANCEIRA 8º ANOMarcosViniciusLemesL
 
Cultura e Sociedade - Texto de Apoio.pdf
Cultura e Sociedade - Texto de Apoio.pdfCultura e Sociedade - Texto de Apoio.pdf
Cultura e Sociedade - Texto de Apoio.pdfaulasgege
 
Programa de Intervenção com Habilidades Motoras
Programa de Intervenção com Habilidades MotorasPrograma de Intervenção com Habilidades Motoras
Programa de Intervenção com Habilidades MotorasCassio Meira Jr.
 
A galinha ruiva sequencia didatica 3 ano
A  galinha ruiva sequencia didatica 3 anoA  galinha ruiva sequencia didatica 3 ano
A galinha ruiva sequencia didatica 3 anoandrealeitetorres
 
ALMANANHE DE BRINCADEIRAS - 500 atividades escolares
ALMANANHE DE BRINCADEIRAS - 500 atividades escolaresALMANANHE DE BRINCADEIRAS - 500 atividades escolares
ALMANANHE DE BRINCADEIRAS - 500 atividades escolaresLilianPiola
 
Modelos de Desenvolvimento Motor - Gallahue, Newell e Tani
Modelos de Desenvolvimento Motor - Gallahue, Newell e TaniModelos de Desenvolvimento Motor - Gallahue, Newell e Tani
Modelos de Desenvolvimento Motor - Gallahue, Newell e TaniCassio Meira Jr.
 
A Arte de Escrever Poemas - Dia das Mães
A Arte de Escrever Poemas - Dia das MãesA Arte de Escrever Poemas - Dia das Mães
A Arte de Escrever Poemas - Dia das MãesMary Alvarenga
 
Slides Lição 03, Central Gospel, O Arrebatamento, 1Tr24.pptx
Slides Lição 03, Central Gospel, O Arrebatamento, 1Tr24.pptxSlides Lição 03, Central Gospel, O Arrebatamento, 1Tr24.pptx
Slides Lição 03, Central Gospel, O Arrebatamento, 1Tr24.pptxLuizHenriquedeAlmeid6
 
ABRIL VERDE.pptx Slide sobre abril ver 2024
ABRIL VERDE.pptx Slide sobre abril ver 2024ABRIL VERDE.pptx Slide sobre abril ver 2024
ABRIL VERDE.pptx Slide sobre abril ver 2024Jeanoliveira597523
 
Bingo da potenciação e radiciação de números inteiros
Bingo da potenciação e radiciação de números inteirosBingo da potenciação e radiciação de números inteiros
Bingo da potenciação e radiciação de números inteirosAntnyoAllysson
 
QUARTA - 1EM SOCIOLOGIA - Aprender a pesquisar.pptx
QUARTA - 1EM SOCIOLOGIA - Aprender a pesquisar.pptxQUARTA - 1EM SOCIOLOGIA - Aprender a pesquisar.pptx
QUARTA - 1EM SOCIOLOGIA - Aprender a pesquisar.pptxIsabellaGomes58
 
A experiência amorosa e a reflexão sobre o Amor.pptx
A experiência amorosa e a reflexão sobre o Amor.pptxA experiência amorosa e a reflexão sobre o Amor.pptx
A experiência amorosa e a reflexão sobre o Amor.pptxfabiolalopesmartins1
 
trabalho wanda rocha ditadura
trabalho wanda rocha ditaduratrabalho wanda rocha ditadura
trabalho wanda rocha ditaduraAdryan Luiz
 
Habilidades Motoras Básicas e Específicas
Habilidades Motoras Básicas e EspecíficasHabilidades Motoras Básicas e Específicas
Habilidades Motoras Básicas e EspecíficasCassio Meira Jr.
 
Prática de interpretação de imagens de satélite no QGIS
Prática de interpretação de imagens de satélite no QGISPrática de interpretação de imagens de satélite no QGIS
Prática de interpretação de imagens de satélite no QGISVitor Vieira Vasconcelos
 
HORA DO CONTO3_BECRE D. CARLOS I_2023_2024
HORA DO CONTO3_BECRE D. CARLOS I_2023_2024HORA DO CONTO3_BECRE D. CARLOS I_2023_2024
HORA DO CONTO3_BECRE D. CARLOS I_2023_2024Sandra Pratas
 
William J. Bennett - O livro das virtudes para Crianças.pdf
William J. Bennett - O livro das virtudes para Crianças.pdfWilliam J. Bennett - O livro das virtudes para Crianças.pdf
William J. Bennett - O livro das virtudes para Crianças.pdfAdrianaCunha84
 

Último (20)

LEMBRANDO A MORTE E CELEBRANDO A RESSUREIÇÃO
LEMBRANDO A MORTE E CELEBRANDO A RESSUREIÇÃOLEMBRANDO A MORTE E CELEBRANDO A RESSUREIÇÃO
LEMBRANDO A MORTE E CELEBRANDO A RESSUREIÇÃO
 
HORA DO CONTO5_BECRE D. CARLOS I_2023_2024
HORA DO CONTO5_BECRE D. CARLOS I_2023_2024HORA DO CONTO5_BECRE D. CARLOS I_2023_2024
HORA DO CONTO5_BECRE D. CARLOS I_2023_2024
 
Investimentos. EDUCAÇÃO FINANCEIRA 8º ANO
Investimentos. EDUCAÇÃO FINANCEIRA 8º ANOInvestimentos. EDUCAÇÃO FINANCEIRA 8º ANO
Investimentos. EDUCAÇÃO FINANCEIRA 8º ANO
 
Cultura e Sociedade - Texto de Apoio.pdf
Cultura e Sociedade - Texto de Apoio.pdfCultura e Sociedade - Texto de Apoio.pdf
Cultura e Sociedade - Texto de Apoio.pdf
 
Programa de Intervenção com Habilidades Motoras
Programa de Intervenção com Habilidades MotorasPrograma de Intervenção com Habilidades Motoras
Programa de Intervenção com Habilidades Motoras
 
A galinha ruiva sequencia didatica 3 ano
A  galinha ruiva sequencia didatica 3 anoA  galinha ruiva sequencia didatica 3 ano
A galinha ruiva sequencia didatica 3 ano
 
ALMANANHE DE BRINCADEIRAS - 500 atividades escolares
ALMANANHE DE BRINCADEIRAS - 500 atividades escolaresALMANANHE DE BRINCADEIRAS - 500 atividades escolares
ALMANANHE DE BRINCADEIRAS - 500 atividades escolares
 
Modelos de Desenvolvimento Motor - Gallahue, Newell e Tani
Modelos de Desenvolvimento Motor - Gallahue, Newell e TaniModelos de Desenvolvimento Motor - Gallahue, Newell e Tani
Modelos de Desenvolvimento Motor - Gallahue, Newell e Tani
 
A Arte de Escrever Poemas - Dia das Mães
A Arte de Escrever Poemas - Dia das MãesA Arte de Escrever Poemas - Dia das Mães
A Arte de Escrever Poemas - Dia das Mães
 
Slides Lição 03, Central Gospel, O Arrebatamento, 1Tr24.pptx
Slides Lição 03, Central Gospel, O Arrebatamento, 1Tr24.pptxSlides Lição 03, Central Gospel, O Arrebatamento, 1Tr24.pptx
Slides Lição 03, Central Gospel, O Arrebatamento, 1Tr24.pptx
 
ABRIL VERDE.pptx Slide sobre abril ver 2024
ABRIL VERDE.pptx Slide sobre abril ver 2024ABRIL VERDE.pptx Slide sobre abril ver 2024
ABRIL VERDE.pptx Slide sobre abril ver 2024
 
Bingo da potenciação e radiciação de números inteiros
Bingo da potenciação e radiciação de números inteirosBingo da potenciação e radiciação de números inteiros
Bingo da potenciação e radiciação de números inteiros
 
Orientação Técnico-Pedagógica EMBcae Nº 001, de 16 de abril de 2024
Orientação Técnico-Pedagógica EMBcae Nº 001, de 16 de abril de 2024Orientação Técnico-Pedagógica EMBcae Nº 001, de 16 de abril de 2024
Orientação Técnico-Pedagógica EMBcae Nº 001, de 16 de abril de 2024
 
QUARTA - 1EM SOCIOLOGIA - Aprender a pesquisar.pptx
QUARTA - 1EM SOCIOLOGIA - Aprender a pesquisar.pptxQUARTA - 1EM SOCIOLOGIA - Aprender a pesquisar.pptx
QUARTA - 1EM SOCIOLOGIA - Aprender a pesquisar.pptx
 
A experiência amorosa e a reflexão sobre o Amor.pptx
A experiência amorosa e a reflexão sobre o Amor.pptxA experiência amorosa e a reflexão sobre o Amor.pptx
A experiência amorosa e a reflexão sobre o Amor.pptx
 
trabalho wanda rocha ditadura
trabalho wanda rocha ditaduratrabalho wanda rocha ditadura
trabalho wanda rocha ditadura
 
Habilidades Motoras Básicas e Específicas
Habilidades Motoras Básicas e EspecíficasHabilidades Motoras Básicas e Específicas
Habilidades Motoras Básicas e Específicas
 
Prática de interpretação de imagens de satélite no QGIS
Prática de interpretação de imagens de satélite no QGISPrática de interpretação de imagens de satélite no QGIS
Prática de interpretação de imagens de satélite no QGIS
 
HORA DO CONTO3_BECRE D. CARLOS I_2023_2024
HORA DO CONTO3_BECRE D. CARLOS I_2023_2024HORA DO CONTO3_BECRE D. CARLOS I_2023_2024
HORA DO CONTO3_BECRE D. CARLOS I_2023_2024
 
William J. Bennett - O livro das virtudes para Crianças.pdf
William J. Bennett - O livro das virtudes para Crianças.pdfWilliam J. Bennett - O livro das virtudes para Crianças.pdf
William J. Bennett - O livro das virtudes para Crianças.pdf
 

Uma palavra aos alunos e professores sobre o Banco de Questões de Matemática

  • 1. 5 Uma palavra aos alunos e professores 9 Uma palavra aos alunos e professores O Banco de Quest˜es (BQ) foi concebido para divulgar nas escolas da rede p´blica material o u de competi¸˜es de Matem´tica, nacionais ou internacionais. Por isso grande parte do conte´do co a u n˜o ´ original, s˜o quest˜es dessas competi¸˜es ou de prepara¸˜o para elas encontradas em a e a o co ca diversos sites e apostilas. Aproveitamos para agradecer a todos que mantˆm esses sites com e livre acesso pela grande contribui¸˜o que d˜o a tantos alunos e professores. ca a Como temos feito desde 2 005, n˜o nos preocupamos com uniformidade. A cada ano o a BQ apresenta formato, quantidade e n´ de dificuldade diferentes dos anos anteriores. A ıvel linguagem usada nas solu¸˜es ´ bastante informal mas sem comprometer o rigor matem´tico. co e a O BQ n˜o ´ um livro did´tico e por isso continuamos a produzi-lo de forma bastante artesanal. a e a Incentivamos alunos e professores a procurar solu¸˜es diferentes das aqui apresentadas, co com certeza elas existem e podem ser mais interessantes. Por solicita¸˜o de muitos alunos, retomamos esse ano a sess˜o Desafios aonde os problemas ca a requerem mais paciˆncia, mais tempo e mais aten¸˜o. Aproveitamos para informar que temos e ca agora no site da OBMEP (www.obmep.org.br ) a sess˜o “Problemas da 15na” com material a muito instigante e desafiador para aqueles que gostam de “quebrar a cabe¸a” com problemas c de Matem´tica. a Os problemas est˜o agrupados em trˆs n´ a e ıveis conforme ´ feito nas provas da OBMEP, mas e muitos s˜o interessantes para todos os alunos. a Sugest˜es quaisquer (por exemplo, de solu¸˜es diferentes) ou cr´ o co ıticas ser˜o bem recebidas a no email: contato@obmep.org.br Desejamos que esse Banco de Quest˜es proporcione a todos bons momentos de reflex˜o e o a descobertas. Dire¸˜o Acadˆmica da OBMEP ca e OBMEP 2009 i
  • 2. 5 Uma palavra aos alunos e professores 9 Organizado por: • Suely Druck (UFF) • Maria Elasir Seabra Gomes (UFMG) Com a colabora¸˜o de: ca • Ana Catarina P. Hellmeister (USP/SP) • F´bio Brochero (UFMG) a • Francisco Dutenhefner (UFMG) Texto j´ revisado pela nova ortografia. a ii OBMEP 2009
  • 3. Conte´do u Uma palavra aos alunos e professores i N´ 1 ıvel 1 Lista 1 . . . . . . . . . . . . . . . . . . . . . . . . . . . . . . . . . . . . . . . . . . 1 Lista 2 . . . . . . . . . . . . . . . . . . . . . . . . . . . . . . . . . . . . . . . . . . 2 Lista 3 . . . . . . . . . . . . . . . . . . . . . . . . . . . . . . . . . . . . . . . . . . 3 Lista 4 . . . . . . . . . . . . . . . . . . . . . . . . . . . . . . . . . . . . . . . . . . 4 Lista 5 . . . . . . . . . . . . . . . . . . . . . . . . . . . . . . . . . . . . . . . . . . 5 Lista 6 . . . . . . . . . . . . . . . . . . . . . . . . . . . . . . . . . . . . . . . . . . 6 Lista 7 . . . . . . . . . . . . . . . . . . . . . . . . . . . . . . . . . . . . . . . . . . 7 Lista 8 . . . . . . . . . . . . . . . . . . . . . . . . . . . . . . . . . . . . . . . . . . 8 Lista 9 . . . . . . . . . . . . . . . . . . . . . . . . . . . . . . . . . . . . . . . . . . 9 Lista 10 . . . . . . . . . . . . . . . . . . . . . . . . . . . . . . . . . . . . . . . . . 10 N´ 2 ıvel 11 Lista 1 . . . . . . . . . . . . . . . . . . . . . . . . . . . . . . . . . . . . . . . . . . 11 Lista 2 . . . . . . . . . . . . . . . . . . . . . . . . . . . . . . . . . . . . . . . . . . 12 Lista 3 . . . . . . . . . . . . . . . . . . . . . . . . . . . . . . . . . . . . . . . . . . 13 Lista 4 . . . . . . . . . . . . . . . . . . . . . . . . . . . . . . . . . . . . . . . . . . 14 Lista 5 . . . . . . . . . . . . . . . . . . . . . . . . . . . . . . . . . . . . . . . . . . 15 Lista 6 . . . . . . . . . . . . . . . . . . . . . . . . . . . . . . . . . . . . . . . . . . 16 Lista 7 . . . . . . . . . . . . . . . . . . . . . . . . . . . . . . . . . . . . . . . . . . 17 Lista 8 . . . . . . . . . . . . . . . . . . . . . . . . . . . . . . . . . . . . . . . . . . 18 Lista 9 . . . . . . . . . . . . . . . . . . . . . . . . . . . . . . . . . . . . . . . . . . 19 Lista 10 . . . . . . . . . . . . . . . . . . . . . . . . . . . . . . . . . . . . . . . . . 20 N´ 3 ıvel 21 Lista 1 . . . . . . . . . . . . . . . . . . . . . . . . . . . . . . . . . . . . . . . . . . 21 Lista 2 . . . . . . . . . . . . . . . . . . . . . . . . . . . . . . . . . . . . . . . . . . 22 Lista 3 . . . . . . . . . . . . . . . . . . . . . . . . . . . . . . . . . . . . . . . . . . 23 Lista 4 . . . . . . . . . . . . . . . . . . . . . . . . . . . . . . . . . . . . . . . . . . 25 Lista 5 . . . . . . . . . . . . . . . . . . . . . . . . . . . . . . . . . . . . . . . . . . 26 Lista 6 . . . . . . . . . . . . . . . . . . . . . . . . . . . . . . . . . . . . . . . . . . 27 Lista 7 . . . . . . . . . . . . . . . . . . . . . . . . . . . . . . . . . . . . . . . . . . 28 Lista 8 . . . . . . . . . . . . . . . . . . . . . . . . . . . . . . . . . . . . . . . . . . 29 Lista 9 . . . . . . . . . . . . . . . . . . . . . . . . . . . . . . . . . . . . . . . . . . 30 iii
  • 4. 5 Uma palavra aos alunos e professores 9 Lista 10 . . . . . . . . . . . . . . . . . . . . . . . . . . . . . . . . . . . . . . . . . 31 Desafios 32 Solu¸˜es do N´ 1 co ıvel 35 Lista 1 . . . . . . . . . . . . . . . . . . . . . . . . . . . . . . . . . . . . . . . . . . 35 Lista 2 . . . . . . . . . . . . . . . . . . . . . . . . . . . . . . . . . . . . . . . . . . 38 Lista 3 . . . . . . . . . . . . . . . . . . . . . . . . . . . . . . . . . . . . . . . . . . 40 Lista 4 . . . . . . . . . . . . . . . . . . . . . . . . . . . . . . . . . . . . . . . . . . 43 Lista 5 . . . . . . . . . . . . . . . . . . . . . . . . . . . . . . . . . . . . . . . . . . 46 Lista 6 . . . . . . . . . . . . . . . . . . . . . . . . . . . . . . . . . . . . . . . . . . 49 Lista 7 . . . . . . . . . . . . . . . . . . . . . . . . . . . . . . . . . . . . . . . . . . 53 Lista 8 . . . . . . . . . . . . . . . . . . . . . . . . . . . . . . . . . . . . . . . . . . 55 Lista 9 . . . . . . . . . . . . . . . . . . . . . . . . . . . . . . . . . . . . . . . . . . 57 Lista 10 . . . . . . . . . . . . . . . . . . . . . . . . . . . . . . . . . . . . . . . . . 59 Solu¸˜es do N´ 2 co ıvel 61 Lista 1 . . . . . . . . . . . . . . . . . . . . . . . . . . . . . . . . . . . . . . . . . . 61 Lista 2 . . . . . . . . . . . . . . . . . . . . . . . . . . . . . . . . . . . . . . . . . . 63 Lista 3 . . . . . . . . . . . . . . . . . . . . . . . . . . . . . . . . . . . . . . . . . . 66 Lista 4 . . . . . . . . . . . . . . . . . . . . . . . . . . . . . . . . . . . . . . . . . . 69 Lista 5 . . . . . . . . . . . . . . . . . . . . . . . . . . . . . . . . . . . . . . . . . . 72 Lista 6 . . . . . . . . . . . . . . . . . . . . . . . . . . . . . . . . . . . . . . . . . . 74 Lista 7 . . . . . . . . . . . . . . . . . . . . . . . . . . . . . . . . . . . . . . . . . . 78 Lista 8 . . . . . . . . . . . . . . . . . . . . . . . . . . . . . . . . . . . . . . . . . . 81 Lista 9 . . . . . . . . . . . . . . . . . . . . . . . . . . . . . . . . . . . . . . . . . . 84 Lista 10 . . . . . . . . . . . . . . . . . . . . . . . . . . . . . . . . . . . . . . . . . 86 Solu¸˜es do N´ 3 co ıvel 88 Lista 1 . . . . . . . . . . . . . . . . . . . . . . . . . . . . . . . . . . . . . . . . . . 88 Lista 2 . . . . . . . . . . . . . . . . . . . . . . . . . . . . . . . . . . . . . . . . . . 91 Lista 3 . . . . . . . . . . . . . . . . . . . . . . . . . . . . . . . . . . . . . . . . . . 93 Lista 4 . . . . . . . . . . . . . . . . . . . . . . . . . . . . . . . . . . . . . . . . . . 96 Lista 5 . . . . . . . . . . . . . . . . . . . . . . . . . . . . . . . . . . . . . . . . . . 100 Lista 6 . . . . . . . . . . . . . . . . . . . . . . . . . . . . . . . . . . . . . . . . . . 103 Lista 7 . . . . . . . . . . . . . . . . . . . . . . . . . . . . . . . . . . . . . . . . . . 106 Lista 8 . . . . . . . . . . . . . . . . . . . . . . . . . . . . . . . . . . . . . . . . . . 110 Lista 9 . . . . . . . . . . . . . . . . . . . . . . . . . . . . . . . . . . . . . . . . . . 114 Lista 10 . . . . . . . . . . . . . . . . . . . . . . . . . . . . . . . . . . . . . . . . . 118 Solu¸˜es dos Desafios co 120 iv OBMEP 2009
  • 5. 5 Lista 1 N´ 1 ıvel 9 N´ 1 ıvel Lista 1 1. Encontro de amigos – Embora eu esteja certo de que meu rel´gio est´ adiantado 5 o a minutos, ele est´, na realidade, com 10 minutos de atraso. Por outro lado, o rel´gio do a o meu amigo est´ realmente 5 minutos adiantado, embora ele pense que est´ correto. N´s a a o marcamos um encontro `s 10 horas e planejamos chegar pontualmente. Quem chegar´ a a em primeiro lugar? Depois de quanto tempo chegar´ o outro? a 2. Trabalho comunit´rio – Uma classe tem 22 alunos e 18 alunas. Durante as f´rias, 60% a e dos alunos dessa classe foram prestar trabalho comunit´rio. No m´ a ınimo, quantas alunas participaram desse trabalho? (A) 1 (B) 2 (C) 4 (D) 6 (E) 8 ´ 3. Area de trap´zios – Unindo quatro trap´zios e e iguais de bases 30 cm e 50 cm e lados n˜o pa- a ralelos iguais, como o da figura, podemos formar um quadrado de ´rea 2 500 cm2 , com um “buraco” a quadrado no meio. Qual ´ a ´rea de cada trap´zio, e a e em cm2 ? (A) 200 (B) 250 (C) 300 (D) 350 (E) 400 4. Adivinha¸˜o – Pensei em 2 n´meros de dois algarismos, que n˜o possuem algarismos ca u a em comum, sendo um o dobro do outro. Al´m disso, os algarismos do menor n´mero e u s˜o a soma e a diferen¸a dos algarismos do maior n´mero. Quais s˜o os n´meros? a c u a u 5. 18 n´meros consecutivos – Escreva 18 n´meros consecutivos de 3 algarismos e veri- u u fique que um deles ´ divis´ pela soma de seus algarismos. e ıvel Isso ´ sempre verdade. Ou seja: se vocˆ escrever 18 n´meros consecutivos de 3 algaris- e e u mos, ent˜o um deles ´ divis´ pela soma de seus algarismos. Mostre este fato. a e ıvel OBMEP 2009 1
  • 6. 5 N´ 1 ıvel Lista 2 9 Lista 2 1. Completar uma tabela – Descubra a regra utilizada para as casas j´ preenchidas e a complete a tabela. Qual ´ o valor de A? e 0 1 2 3 4 1 2 5 10 2 3 4 A 2. Procurando m´ltiplos de 9 – Consideremos um conjunto formado por 10 n´meros u u naturais diferentes. Se calculamos todas as diferen¸as entre esses n´meros, pelo menos c u uma dessas diferen¸as ´ um m´ltiplo de 9? c e u 3. Correndo numa pra¸a – Um atleta costuma correr c 15, 5 km ao redor de uma pra¸a retangular de dimens˜es c o 900 m × 600 m. Ele inicia a corrida sempre do ponto P situado a 550 m de um dos v´rtices correndo no sentido e hor´rio, como mostra a figura. Em que ponto da pra¸a a c ele para? 4. Ovos para um bolo – Uma doceira foi ao mercado comprar ovos para fazer 43 bolos, todos com a mesma receita, que gasta menos de 9 ovos. O vendedor repara que se tentar embrulhar os ovos que a doceira comprou em grupos de 2 ou de 3 ou de 4 ou de 5 ou de 6 ovos, sempre sobra 1 ovo. Quantos ovos ela usa em cada bolo? Qual o menor n´mero de ovos que a doceira vai gastar para fazer os 43 bolos? u 5. C´lculos H e V – Vocˆ consegue colocar os n´meros de 1 a 8 a e u m÷ m m = dentro dos c´ ırculos, sem repeti-los, de modo que os c´lculos na a − × horizontal e na vertical sejam corretos? m m .................................... .................................... Dica: Quais as possibilidades para a multiplica¸˜o? Quais os ca m+ m m = poss´ıveis lugares para o n´mero 1? u 2 OBMEP 2009
  • 7. 5 Lista 3 N´ 1 ıvel 9 Lista 3 1. Cortando uma cartolina – Uma folha retangular de cartolina foi cortada ao longo de sua diagonal. Num dos peda¸os obtidos, foram feitos 2 cortes paralelos aos 2 lados c menores e pelos pontos m´dios desses lados. Ao final sobrou um retˆngulo de per´ e a ımetro 129 cm. O desenho abaixo indica a sequˆncia de cortes. e ...................................................................................................................... . . . . ........ . . . . . . . ........ . . . . . . . . ........ . . . . . . . . ........ . . . . . . . ........ . . . ............................................................ . . . . . . . . . . ............ . . . . . . . . . . . . . . ........ . . . . . . . . . . . . . . . . - .... .... ........ . . . . . . . - . . . . . . . . . . . . . . . . . . . . . ........ .... . . . . . . . . . . . . . . . . . . ............ . . . . . . . . . . . . . . . . . . . . ............ . . . . ............................................................ . . . . . . . . ........ ... . . . . . . . . ...................................................................................................................... . . .. ............. ................................................................................................................... . . . Qual era o per´ ımetro da folha antes do corte? 2. A soma errada – A soma ao lado est´ incorreta. Para corrigi-la basta a 742586 substituir um certo algarismo em todos os lugares que ele aparece na conta +829430 por um outro algarismo. Quais s˜o esses dois algarismos? a 1212016 3. N´mero de 5 algarismos – Os algarismos 1, 2, 3, 4 e 5 foram usados, cada um uma u unica vez, para escrever um n´mero de 5 algarismos a b c d e, tal que: a b c ´ divis´ ´ u e ıvel por 4, b c d por 5, e c d e por 3. Encontre esse n´mero. u 4. Tabela misteriosa – Complete a tabela 6 × 6 de modo 32 40 que em cada linha e cada coluna apare¸am apenas c 49 m´ltiplos de um dos n´meros: u u 22 15 2, 3, 4, 5, 6, 7, 8, 9, 10, 11, 12 . 24 42 Vocˆ pode repetir apenas um n´mero na tabela. e u 5. Habitantes e esporte – Numa cidade com quase 30 mil habitantes, dois nonos dos homens e dois quinze avos das mulheres pratica esporte somente nos finais de semana, e o n´mero de habitantes que n˜o pratica esporte ´ o qu´ u a e ıntuplo dos que praticam esporte regularmente. Com esses dados, complete a tabela. N˜o praticam esporte a Praticam esporte somente Praticam esporte Popula¸˜o ca nos finais de semana regularmente fem. masc. fem. masc. fem. masc. total 8 563 8 322 1 252 OBMEP 2009 3
  • 8. 5 N´ 1 ıvel Lista 4 9 Lista 4 PSfrag replacements 1. Bot˜es luminosos – No mecanismo luminoso da figura, o 1 1 cada um dos oito bot˜es pode acender as cores verde ou o 2 8 2 azul. O mecanismo funciona do seguinte modo: ao ser 3 ligado, todos os bot˜es acendem a luz azul, e se aperta- o 4 3 mos um bot˜o, esse bot˜o e seus vizinhos trocam de cor. a a 7 5 Se ligarmos o mecanismo e apertarmos sucessivamente os 6 bot˜es 1, 3 e 5, qual ser´ o n´mero de luzes verdes que o a u 6 4 7 estar˜o acesas no final? a 5 8 (A) 3 (B) 4 (C) 5 (D) 6 (E) 7 2. Qual ´ o n´mero? – Um n´mero de 6 algarismos come¸a por 1. Se deslocamos esse e u u c algarismo 1 da primeira posi¸˜o para a ultima ` direita, obtemos um novo n´mero de 6 ca ´ a u algarismos que ´ o triplo do n´mero de partida. Qual ´ esse n´mero? e u e u 3. Jardim variado – Um jardim retangular de 120 m por 80 m foi dividido em 6 regi˜es o como na figura, onde N, M e P s˜o pontos m´dios dos lados, e R divide o comprimento a e na raz˜o 1/3. Em cada regi˜o ser´ plantado um dos seguintes tipos de flor: rosa, a a a margarida, cravo, bem-me-quer, violeta e brom´lia, cujos pre¸os, por m2 est˜o indicados e c a na tabela. Quais as poss´ ıveis escolhas das flores em cada regi˜o, de modo a gastar o a m´ınimo poss´ ıvel? Tipo Pre¸o por m2 c rosa 3,50 margarida 1,20 cravo 2,20 bem-me-quer 0,80 violeta 1,70 brom´lia e 3,00 4. O algarismo 3 – Luis escreveu a sequˆncia de n´meros naturais a partir de 1: e u 1, 2, 3, 4, 5, 6, 7, 8, 9, 10, 11, 12, · · · . Quando ele escreveu o algarismo 3 pela 25a vez? 5. Soma de potˆncias – O n´mero 3444 + 4333 ´ divis´ por 5? e u e ıvel 4 OBMEP 2009
  • 9. 5 Lista 5 N´ 1 ıvel 9 Lista 5 1. Telefonemas – Jo˜o mora em Salvador e seus pais em Recife. Para matar a saudade, a ele telefona para seus pais a cada trˆs dias. O primeiro telefonema foi feito no domingo, e o segundo telefonema na 4a feira, o terceiro telefonema no s´bado, e assim por diante. a Em qual dia da semana Jo˜o telefonou para seus pais pela cent´sima vez? a e 2. O maior produto – Com os algarismos de 1 a 5 e um sinal X 1 2 de multiplica¸˜o × Clara forma o produto de 2 n´meros, com ca u o sinal × entre eles. Como Clara deve colocar os cart˜es para o 3 4 5 obter o maior produto poss´ıvel? P 3. O caminho da Joaninha – Dona Joaninha quer atraves- sar um p´tio com azulejos quadrados numerados como a mostra a figura. Ela vai partir do ponto P e quer chegar ao ponto C andando somente sobre os lados dos azule- jos. Dona Joaninha n˜o quer ter n´meros primos ` sua a u a direita ao longo de todo o percurso. Qual ´ o menor e percurso que ela pode fazer? C ................................ ........ ..... ..... 4. O lugar dos amigos – Sete amigos tra¸aram um triˆngulo, um c a .. . .... . . . ... ................................................... ... .. .. . . . quadrado e um c´ ırculo. Cada um marcou seu lugar com um n´mero: u .. . . . 1 . . . . ... . . . . .... ..... 2 . . . . . . . . . . . . .. .. ..... . ... .... . . . .... .. . . . . 7 . . . . . . ... .... ...... . . . . . . 3 ........ . . . . . . . . . 4 ..... ... . . . . .......... . .......... . . .. ................... 6 ..... ...... .... . . . . . . . Ana: “Eu n˜o falarei nada.” a .... . .... 5 . . .... . . .................................................... .... ............................................................................ .. Bento: “Eu estou dentro de uma unica figura.” ´ Celina: “Eu estou dentro das trˆs figuras.” e Diana: “Eu estou dentro do triˆngulo mas n˜o do quadrado.” a a Elisa: “Eu estou dentro do triˆngulo e do c´ a ırculo.” F´bio: a “Eu n˜o estou dentro de um pol´ a ıgono.” Guilherme: “Eu estou dentro do c´ırculo.” Encontre o lugar de cada um. 5. Quadrado perfeito? – Cada um dos cinco n´meros abaixo tem 100 algarismos, e ´ u e formado pela repeti¸˜o de um ou dois algarismos: ca N1 = 333333 . . . 3 N2 = 666666 . . . 6 N3 = 151515 . . . 15 N4 = 212121 . . . 21 N5 = 272727 . . . 27 Algum destes n´meros ´ um quadrado perfeito? u e OBMEP 2009 5
  • 10. 5 N´ 1 ıvel Lista 6 9 Lista 6 1. Preenchendo quadradinhos – Complete os quadradinhos com os n´meros 1, 2, 3, 5, 6. u + − × ÷ = 4 2. Os 3 n´meros – Sofia brinca de escrever todos os n´meros de 4 algarismos diferentes u u que se pode escrever com os algarismos 1, 2, 4 e 7. Ela soma 3 desses n´meros – todos u diferentes – e obt´m 13 983. Quais s˜o esses 3 n´meros? e a u 3. Preencher uma tabela – Jandira deve preencher uma tabela 4 × 4 que j´ vem com duas casas preenchidas com os n´meros a u 1 e 2 - veja ao lado. Duas casas s˜o consideradas vizinhas se a 1 2 tˆm um v´rtice ou um lado em comum. e e As regras que ela tem que obedecer s˜o: a • uma casa s´ pode ser preenchida se alguma de suas casas vizinhas j´ cont´m um o a e n´mero; u • ao preencher uma casa, deve-se colocar a soma de todos os n´meros que j´ constam u a em suas casas vizinhas. Qual ´ o maior n´mero que ´ poss´ escrever na tabela? e u e ıvel 4. Olimp´ ıada de Pequim – Na Olimp´ ıada de Pequim sentaram-se, em uma mesa quadrada, as mulheres, Maria e Tˆnia, e os homens, Juan e David, todos atletas. Cada um deles a pratica um esporte diferente: nata¸˜o, vˆlei, gin´stica e atletismo. Eles estavam sentados ca o a da seguinte maneira: (a) Quem pratica a nata¸˜o estava ` esquerda de Maria. ca a (b) Quem pratica gin´stica estava em frente a Juan. a (c) Tˆnia e David sentaram-se lado a lado. a (d) Uma mulher sentou-se ao lado de quem pratica volei. Qual dos atletas pratica atletismo? 5. Culturas diferentes – Jorge, que mora em Recife, se corresponde com seu amigo inglˆs e Ralph que mora na Inglaterra. Os dois se compreendem muito bem nas duas l´ ınguas, mas tˆm um problema com as datas: a data 08/10 no Brasil significa 8 de outubro, e na e Inglaterra 10 de agosto. Por causa disso, os dois combinaram n˜o se escrever nos dias a em que a data for amb´ ıgua. Eles preferem datas como 25/03 que s´ pode significar 25 o de mar¸o. c (a) Em quais das datas a seguir Jorge e Ralph n˜o podem se escrever? a (i) 3 de dezembro (ii) 18 de agosto (iii) 5 de maio (b) Quando ocorre o maior per´ ıodo em que os dois amigos n˜o podem se escrever? a 6 OBMEP 2009
  • 11. 5 Lista 7 N´ 1 ıvel 9 Lista 7 1. Uma liquida¸˜o – Na liquida¸˜o da loja SUPER-SUPER todos os produtos est˜o 50% ca ca a mais baratos, e aos s´bados existe ainda um desconto adicional de 20%. Carla com- a prou uma cal¸a antes da liquida¸˜o, e agora ela se lamenta: Nesse s´bado eu teria c ca a economizado R$ 50,40 na cal¸a. Qual era o pre¸o da cal¸a antes da liquida¸˜o? c c c ca 2. N´mero com muitos zeros – Se a ´ o n´mero 0, 000 . . . 000 1, ent˜o qual das express˜es u e u a o 2009 zeros a seguir representa o maior n´mero? u (A) 3+a (B) 3−a (C) 3a (D) 3/a (E) a/3 3. Corrida das tartarugas – Cinco tartarugas apostaram uma corrida em linha reta e na chegada a situa¸˜o foi a seguinte: Sininha est´ 10 m atr´s de Olguinha e 25 m ` frente ca a a a de Rosinha que est´ 5 m atr´s de Elzinha que est´ 25 m atr´s de Pulinha. Qual foi a a a a a ordem de chegada? 4. Que mem´ria... – Esquecinaldo tem p´ssima mem´ria para guardar n´meros, mas o e o u o ´tima para lembrar sequˆncias de opera¸˜es. Por isso, para lembrar do seu c´digo e co o banc´rio de 5 algarismos, ele consegue se lembrar que nenhum dos algarismos ´ zero, a e os dois primeiros algarismos formam uma potˆncia de 5, os dois ultimos formam uma e ´ potˆncia de 2, o do meio ´ um m´ltiplo de 3 e a soma de todos os algarismos ´ um e e u e n´mero ´ u ımpar. Agora ele n˜o precisa mais decorar o n´mero porque ele sabe que ´ o a u e maior n´mero que satisfaz essas condi¸˜es e que n˜o tem algarismos repetidos. Qual ´ u co a e esse c´digo? o 5. Uma fra¸˜o irredut´ ca ıvel – Encontre uma fra¸˜o irredut´ tal que o produto de seu ca ıvel numerador pelo denominador seja 2 × 3 × 4 × 5 × . . . × 10. Quantas dessas fra¸˜es co irredut´ ıveis existem? OBMEP 2009 7
  • 12. 5 N´ 1 ıvel Lista 8 9 Lista 8 1. Transformar em decimal – Escreva o resultado das seguintes express˜es na forma o decimal: 2 5 5 2 (a) 7× + 16 × (b) 5− 2÷ (c) 1+ 3 3 12 3 1+ 1+4 2. Uma sequˆncia especial – Escrevendo sucessivamente os n´meros naturais, obtemos e u a sequˆncia: e 1 2 3 4 5 6 7 8 9 10 11 12 13 14 15 16 17 18 19 20 21 22 . . . Qual algarismo est´ na 2 009a posi¸˜o dessa sequˆncia? a ca e 3. Cortar um retˆngulo – Como cortar um retˆngulo de 13 cm por 7 cm em 13 retˆngulos a a a diferentes? 4. Medida de ˆngulo – Na figura, AOD e B OY s˜o ˆngulos retos e a medida de DOY a a a est´ entre 40◦ e 50◦ . Al´m disso, os pontos C e Y est˜o sobre a reta r, enquanto a e a D e E est˜o sobre a reta s. Os poss´ a ıveis valores para a medida de AOC variam de: (A) 30◦ a 40◦ (B) 40◦ a 50◦ (C) 50◦ a 60◦ (D) 40◦ a 60◦ (E) n˜o podem ser determinados a √ 5. Per´ımetros e ´reas – Um quadrado tem 3 √ 3 cm de lado, e as dimens˜es de um a √ √ + o ımetros, s˜o 72 + 3 6 e 2. Qual dos dois tem maior ´rea? E retˆngulo, em cent´ a a a maior per´ ımetro? 6. C´lculo de ˆngulo – Encontre B AD, sabendo a a que DAC = 39◦ , AB = AC e AD = BD. 8 OBMEP 2009
  • 13. 5 Lista 9 N´ 1 ıvel 9 Lista 9 1. O caminho da formiga – Uma formiga sai de um ponto A, anda 7 cm para a esquerda, 5 cm para cima, 3 cm para a direita, 2 cm para baixo, 9 cm para a direita, 2 cm para baixo, 1 cm para a esquerda e 1 cm para baixo, chegando no ponto B. Qual ´ a distˆncia e a d entre A e B? (A) 0 cm (B) 1 cm (C) 4 cm (D) 5 cm (E) 7 cm 2. Menino mentiroso – Jo˜ozinho mente nas ter¸as-feiras, quintas-feiras e s´bados e o a c a resto dos dias fala a verdade. Um dia Pedrinho encontra com Jo˜ozinho e tˆm o seguinte a e di´logo: a • Pedrinho pergunta: Que dia ´ hoje? e • Jo˜ozinho responde: S´bado. a a • Pedrinho pergunta: E que dia ser´ amanh˜? a a • Jo˜ozinho responde: Quarta-feira. a Que dia da semana o Pedrinho encontrou com o Jo˜ozinho? a 3. Encontre os 4 n´meros – Encontre quatro n´meros distintos de 3 algarismos, tais que u u a soma de trˆs quaisquer deles ´ divis´ pelo quarto n´mero. e e ıvel u 4. Colando 6 triˆngulos – Construa 6 triˆngulos a a equil´teros, o primeiro com lado de comprimento 1 cm e a os triˆngulos seguintes com lado igual a metade do lado a do triˆngulo anterior, como indicado na figura ao lado. a Qual ´ o per´ e ımetro desta figura? 5. Os livros da Elisa – Elisa tem 24 livros de ciˆncias e outros de matem´tica e litera- e a 1 tura. Se Elisa tivesse um livro a mais de matem´tica, ent˜o de seus livros seria de a a 9 matem´tica e um quarto de literatura. Se Elisa tem menos que 100 livros, quantos livros a de matem´tica ela possui? a OBMEP 2009 9
  • 14. 5 N´ 1 ıvel Lista 10 9 Lista 10 1. Divis˜o por 9 – a (a) Listemos os primeiros 20 092 009 n´meros naturais. Em seguida, substitu´ u ımos, sucessivamente, cada n´mero pela soma dos seus algarismos, at´ obtermos uma u e lista de n´meros com apenas um algarismo. A lista tem mais algarismos 4 ou 5? u Quantos 9 tem a lista? (b) Aplicando o mesmo processo ao n´mero 32 009 , isto ´, substituindo o n´mero pela u e u soma dos seus algarismos, qual ´ o n´mero de apenas um algarismo obtido? e u (c) E para o n´mero 172 009 ? u 2. Uma brincadeira na sala de aula – A professora Raquel inventou a seguinte brincadeira: escreva um n´mero no quadro, se ele for ´ u ımpar acrescente 3 unidades ao n´mero, e se u ele for par divida o n´mero por 2. u Esta opera¸˜o pode ser feita diversas vezes. A professora est´ interessada em obter no ca a final o n´mero 1 e perguntou para a classe: Como obter o n´mero 1 ap´s 3 opera¸˜es? u u o co E ap´s 4 opera¸˜es? E ap´s 5 opera¸˜es? o co o co 3. Calcule a idade – Laura e sua av´ Ana acabaram de descobrir que, no ano passado, o suas idades eram divis´ ıveis por 8 e, no pr´ximo ano, ser˜o divis´ o a ıveis por 7. Vov´ Ana o ainda n˜o ´ centen´ria. Qual ´ a idade de Laura? a e a e 4. Divis˜es e restos – O dobro de um n´mero dividido por 5 deixa resto 1. Qual o resto o u da divis˜o desse n´mero por 5? a u 5. Preenchendo o c´ ırculo – Cada um dos sinais , , , e representa um n´mero u de 1 algarismo. Descubra quem s˜o eles e complete o n´mero que falta no c´ a u ırculo em branco. × × / × + 47 − − 423 − − − 282 − − −→ − −→ −→ − − → 1448 −− 10 OBMEP 2009